Basal Cell Carcinoma: Analysis of Factors Associated With Incomplete Excision at a Referral Hospital in Southern Spain

Article Type
Changed
Thu, 01/10/2019 - 13:13
Display Headline
Basal Cell Carcinoma: Analysis of Factors Associated With Incomplete Excision at a Referral Hospital in Southern Spain

Article PDF
Author and Disclosure Information

Husein Husein-ElAhmed, MD; Jose Aneiros-Fernandez, MD; Maria-Teresa Gutierrez-Salmeron, MD; Jose Aneiros-Cachaza, MD; Ramon Naranjo-Sintes, MD

Issue
Cutis - 93(3)
Publications
Topics
Page Number
155-161
Legacy Keywords
basal cell carcinoma, incomplete excision, surgical technique, incomplete excision of nonmelanoma skin cancer, nonmelanoma skin cancer, BCC, treatment modalities for BCC
Author and Disclosure Information

Husein Husein-ElAhmed, MD; Jose Aneiros-Fernandez, MD; Maria-Teresa Gutierrez-Salmeron, MD; Jose Aneiros-Cachaza, MD; Ramon Naranjo-Sintes, MD

Author and Disclosure Information

Husein Husein-ElAhmed, MD; Jose Aneiros-Fernandez, MD; Maria-Teresa Gutierrez-Salmeron, MD; Jose Aneiros-Cachaza, MD; Ramon Naranjo-Sintes, MD

Article PDF
Article PDF
Related Articles

Issue
Cutis - 93(3)
Issue
Cutis - 93(3)
Page Number
155-161
Page Number
155-161
Publications
Publications
Topics
Article Type
Display Headline
Basal Cell Carcinoma: Analysis of Factors Associated With Incomplete Excision at a Referral Hospital in Southern Spain
Display Headline
Basal Cell Carcinoma: Analysis of Factors Associated With Incomplete Excision at a Referral Hospital in Southern Spain
Legacy Keywords
basal cell carcinoma, incomplete excision, surgical technique, incomplete excision of nonmelanoma skin cancer, nonmelanoma skin cancer, BCC, treatment modalities for BCC
Legacy Keywords
basal cell carcinoma, incomplete excision, surgical technique, incomplete excision of nonmelanoma skin cancer, nonmelanoma skin cancer, BCC, treatment modalities for BCC
Article Source

PURLs Copyright

Inside the Article

Article PDF Media
Image
Disable zoom
Off

A skin lesion after cardiac catheterization

Article Type
Changed
Tue, 10/03/2017 - 08:32
Display Headline
A skin lesion after cardiac catheterization

A 64-year-old man with diabetes and hypertension presented with a 2-day history of sudden onset of acute pain and cyanosis on the sole of his right foot 4 days after undergoing cardiac catheterization and coronary angiography.

RTEmagicC_Arias-Santiago_SkinLesion_F1.gif.gif
Figure1. Macular violaceous connecting rings in a net-like pattern compatible with livedo reticularis on the foot.
The physical examination revealed macular, violaceous, connecting rings in a net-like pattern that blanched with pressure and disappeared when the foot was elevated, a presentation compatible with livedo reticularis (Figure 1). Laboratory testing (complete blood cell count, biochemistry panel, coagulation test, and C-reactive protein test) was notable only for eosinophilia.

A few days later, the patient returned with abdominal pain, diarrhea, and acute renal injury with urinary eosinophils (7% of the white blood cells in the urine) and proteinuria.

Q: Which is the most likely diagnosis?

  • Infective endocarditis
  • Pernio (chilblain)
  • Cholesterol crystal embolism
  • Cutaneous small-vessel vasculitis

A: Cholesterol crystal embolism is the correct diagnosis.

Infective endocarditis is an infection of the endocardium, but systemic signs may be present, including cutaneous lesions such as Osler nodes (painful papules on the tips of the fingers and toes) and Janeway lesions (painless macules on the palms and soles). Histologic staining of skin biopsy specimens often shows vasculitis, occasionally with a positive Gram stain. Severe renal injury is not common, and the timing of the acute illness and skin lesion fits better with an acute embolic phenomenon.

Pernio is a form of cold injury, localized in peripheral parts of the body and occurring after exposure to cold temperatures in damp conditions. It usually manifests bilaterally as painful erythematous or purple lesions on the acral areas of the hands and feet, nose, ears, and, rarely, the thighs and buttocks. Pernio most commonly affects women between 20 and 40 years of age. It can be idiopathic or associated with a systemic disease such as systemic lupus erythematosus or Sjögren syndrome.

Cutaneous small-vessel vasculitis is a heterogeneous group of disorders with inflammation and damage of the blood vessels; it may be limited to the skin or it may involve multiple systems. Palpable or nonpalpable purpura and ulceration are common clinical findings, and histologic study shows an inflammatory infiltrate of vessel walls, fibrinoid necrosis, thrombosis, and extravasation of red blood cells.

While this patient’s problems are not consistent with small-vessel vasculitis, the single skin lesion, the timing after the catheterization, and the urinary eosinophils are best explained by cholesterol embolization.

CHOLESTEROL CRYSTAL EMBOLISM

Cholesterol crystal embolism is commonly iatrogenic, a complication of mechanical damage to the arterial walls from vascular surgery or invasive percutaneous procedures. Material dislodged from atheromatous arterial plaques can occlude the small vessels of the feet, producing this syndrome.

The onset of the clinical disease is often delayed for days to weeks after an angiographic procedure.1 Spontaneous hemorrhage, disruption of plaque, or drug therapy with an anticoagulant or a fibrinolytic can precipitate embolization of cholesterol crystals. The source of the emboli is atheromatous plaque in major blood vessels, particularly the abdominal aorta.

 

 

Many organs and systems can be affected

These emboli can affect many organs and systems: eg, the kidneys (causing hypertension and acute renal failure), the muscles (causing myalgias), gastrointestinal organs (causing bleeding, abdominal pain, and bowel infarction), the lungs (causing acute respiratory distress syndrome), the eyes (causing Hollenhorst plaques in retinal arteries), and the central nervous system (causing stroke, confusion, and delirium). Cardiac or central nervous system involvement is associated with a high risk of death.

After angiography, clinical manifestations of cholesterol embolization have been reported in 0.06% to 1.4% of patients,2,3 although the finding of cholesterol emboli is more common in autopsy studies.4

Recognizing skin signs is the key

Cutaneous abnormalities are usually the earliest and often the only clinical manifestation of this syndrome. Findings on the lower limbs include blue toes, cutaneous nodules, and livedo reticularis, affecting the feet and legs and sometimes extending upward to the trunk. Other findings include digital infarcts, ulceration, gangrene, purpura, cyanosis, and splinter hemorrhages in the nail bed.

RTEmagicC_Arias-Santiago_SkinLesion_F2.gif.gif
Figure 2. Skin biopsy showed needle-shaped clefts within the lumen of blood vessels, ie, dissolved cholesterol crystals obstructing small arteries.

In our patient, microscopic study of skin biopsy specimens showed needle-shaped clefts within the lumen of blood vessels—ie, dissolved cholesterol crystals obstructing small arteries (Figure 2).

Biopsy studies of skin lesions are positive in a high percentage of cases, showing dissolved cholesterol crystals within the lumen of blood vessels, especially in the small to large arteries and arterioles of the deep dermis or subcutaneous fat. Deep biopsies and carefully examination are necessary, as emboli tend to be patchily distributed. Early recognition of cutaneous clinical findings is essential to establish the proper diagnosis and treatment.

The diagnostic triad of this disease includes blue toe syndrome, acute or subacute renal failure, and a temporal relation with an inciting event (particularly angiography). But despite these diagnostic criteria,2 the diagnosis is often based on a combination of signs and symptoms specific to end-organ damage and a systemic inflammatory response.3

Histologic confirmation is considered essential to the diagnosis of cholesterol crystal embolism, and as the skin is the most accessible site, skin biopsy provides the best sample for histologic diagnosis.5

Postprocedural embolism of a blood clot, vasculitis, and infective endocarditis are the most important differential diagnoses.6,7

Treatment is supportive, preventive

Treatment is mainly supportive with hemodynamic monitoring, nutritional and metabolic support, mechanical ventilation, and dialysis if necessary. The underlying atherosclerotic disease should be treated aggressively. Prevention of another episode involves modification of traditional risk factors such as serum cholesterol, diabetes, hypertension, and smoking. Additional vascular surgery procedures should be avoided, as they can induce new episodes.

References
  1. Donohue KG, Saap L, Falanga V. Cholesterol crystal embolization: an atherosclerotic disease with frequent and varied cutaneous manifestations. J Eur Acad Dermatol Venereol 2003; 17:504511.
  2. Fukumoto Y, Tsutsui H, Tsuchihashi M, Masumoto A, Takeshita A; Cholesterol Embolism Study (CHEST) Investigators. The incidence and risk factors of cholesterol embolization syndrome, a complication of cardiac catheterization: a prospective study. J Am Coll Cardiol 2003; 42:211216.
  3. Johnson LW, Esente P, Giambartolomei A, et al. Peripheral vascular complications of coronary angioplasty by the femoral and brachial techniques. Cathet Cardiovasc Diagn 1994; 31:165172.
  4. Kronzon I, Saric M. Cholesterol embolization syndrome. Circulation 2010; 122:631641.
  5. Jucgla A, Moreso F, Muniesa C, Moreno A, Vidaller A. Cholesterol embolism: still an unrecognized entity with a high mortality rate. J Am Acad Dermatol 2006; 55:786793.
  6. Maki T, Izumi C, Miyake M, et al. Cholesterol embolism after cardiac catheterization mimicking infective endocarditis. Intern Med 2005; 44:10601063.
  7. Arias-Santiago S, Aneiros-Fernández J, Girón-Prieto MS, Fernández-Pugnaire MA, Naranjo-Sintes R. Palpable purpura. Cleve Clin J Med 2010; 77:205206.
Article PDF
Author and Disclosure Information

Salvador Arias-Santiago, MD, PhD
Department of Dermatology, San Cecilio University Hospital, Department of Dermatology, Baza General Hospital, and Department of Histology, School of Medicine, Granada University, Granada, Spain

Jose Aneiros-Fernández, MD
Department of Pathology, San Cecilio University Hospital, Granada, Spain

Victor Carriel, PhD
Department of Histology, School of Medicine, Granada University, Granada, Spain

Jacinto Orgaz-Molina, MD
Department of Dermatology, San Cecilio University Hospital, Granada, Spain

Miguel Gonález-Andrades, MD, PhD
Department of Histology, School of Medicine, Granada University, Granada, Spain

Agustín Buendía-Eisman, MD, PhD
Department of Dermatology, School of Medicine, Granada University, Granada, Spain

Miguel Alaminos, MD, PhD
Department of Histology, School of Medicine, Granada University, Granada, Spain

Address: Salvador Arias-Santiago, MD, Department of Dermatology, San Cecilio University Hospital, Av Dr. Oloriz 16, Granada 18012, Spain; e-mail salvadorarias@hotmail.es

Issue
Cleveland Clinic Journal of Medicine - 79(6)
Publications
Topics
Page Number
424-426
Sections
Author and Disclosure Information

Salvador Arias-Santiago, MD, PhD
Department of Dermatology, San Cecilio University Hospital, Department of Dermatology, Baza General Hospital, and Department of Histology, School of Medicine, Granada University, Granada, Spain

Jose Aneiros-Fernández, MD
Department of Pathology, San Cecilio University Hospital, Granada, Spain

Victor Carriel, PhD
Department of Histology, School of Medicine, Granada University, Granada, Spain

Jacinto Orgaz-Molina, MD
Department of Dermatology, San Cecilio University Hospital, Granada, Spain

Miguel Gonález-Andrades, MD, PhD
Department of Histology, School of Medicine, Granada University, Granada, Spain

Agustín Buendía-Eisman, MD, PhD
Department of Dermatology, School of Medicine, Granada University, Granada, Spain

Miguel Alaminos, MD, PhD
Department of Histology, School of Medicine, Granada University, Granada, Spain

Address: Salvador Arias-Santiago, MD, Department of Dermatology, San Cecilio University Hospital, Av Dr. Oloriz 16, Granada 18012, Spain; e-mail salvadorarias@hotmail.es

Author and Disclosure Information

Salvador Arias-Santiago, MD, PhD
Department of Dermatology, San Cecilio University Hospital, Department of Dermatology, Baza General Hospital, and Department of Histology, School of Medicine, Granada University, Granada, Spain

Jose Aneiros-Fernández, MD
Department of Pathology, San Cecilio University Hospital, Granada, Spain

Victor Carriel, PhD
Department of Histology, School of Medicine, Granada University, Granada, Spain

Jacinto Orgaz-Molina, MD
Department of Dermatology, San Cecilio University Hospital, Granada, Spain

Miguel Gonález-Andrades, MD, PhD
Department of Histology, School of Medicine, Granada University, Granada, Spain

Agustín Buendía-Eisman, MD, PhD
Department of Dermatology, School of Medicine, Granada University, Granada, Spain

Miguel Alaminos, MD, PhD
Department of Histology, School of Medicine, Granada University, Granada, Spain

Address: Salvador Arias-Santiago, MD, Department of Dermatology, San Cecilio University Hospital, Av Dr. Oloriz 16, Granada 18012, Spain; e-mail salvadorarias@hotmail.es

Article PDF
Article PDF

A 64-year-old man with diabetes and hypertension presented with a 2-day history of sudden onset of acute pain and cyanosis on the sole of his right foot 4 days after undergoing cardiac catheterization and coronary angiography.

RTEmagicC_Arias-Santiago_SkinLesion_F1.gif.gif
Figure1. Macular violaceous connecting rings in a net-like pattern compatible with livedo reticularis on the foot.
The physical examination revealed macular, violaceous, connecting rings in a net-like pattern that blanched with pressure and disappeared when the foot was elevated, a presentation compatible with livedo reticularis (Figure 1). Laboratory testing (complete blood cell count, biochemistry panel, coagulation test, and C-reactive protein test) was notable only for eosinophilia.

A few days later, the patient returned with abdominal pain, diarrhea, and acute renal injury with urinary eosinophils (7% of the white blood cells in the urine) and proteinuria.

Q: Which is the most likely diagnosis?

  • Infective endocarditis
  • Pernio (chilblain)
  • Cholesterol crystal embolism
  • Cutaneous small-vessel vasculitis

A: Cholesterol crystal embolism is the correct diagnosis.

Infective endocarditis is an infection of the endocardium, but systemic signs may be present, including cutaneous lesions such as Osler nodes (painful papules on the tips of the fingers and toes) and Janeway lesions (painless macules on the palms and soles). Histologic staining of skin biopsy specimens often shows vasculitis, occasionally with a positive Gram stain. Severe renal injury is not common, and the timing of the acute illness and skin lesion fits better with an acute embolic phenomenon.

Pernio is a form of cold injury, localized in peripheral parts of the body and occurring after exposure to cold temperatures in damp conditions. It usually manifests bilaterally as painful erythematous or purple lesions on the acral areas of the hands and feet, nose, ears, and, rarely, the thighs and buttocks. Pernio most commonly affects women between 20 and 40 years of age. It can be idiopathic or associated with a systemic disease such as systemic lupus erythematosus or Sjögren syndrome.

Cutaneous small-vessel vasculitis is a heterogeneous group of disorders with inflammation and damage of the blood vessels; it may be limited to the skin or it may involve multiple systems. Palpable or nonpalpable purpura and ulceration are common clinical findings, and histologic study shows an inflammatory infiltrate of vessel walls, fibrinoid necrosis, thrombosis, and extravasation of red blood cells.

While this patient’s problems are not consistent with small-vessel vasculitis, the single skin lesion, the timing after the catheterization, and the urinary eosinophils are best explained by cholesterol embolization.

CHOLESTEROL CRYSTAL EMBOLISM

Cholesterol crystal embolism is commonly iatrogenic, a complication of mechanical damage to the arterial walls from vascular surgery or invasive percutaneous procedures. Material dislodged from atheromatous arterial plaques can occlude the small vessels of the feet, producing this syndrome.

The onset of the clinical disease is often delayed for days to weeks after an angiographic procedure.1 Spontaneous hemorrhage, disruption of plaque, or drug therapy with an anticoagulant or a fibrinolytic can precipitate embolization of cholesterol crystals. The source of the emboli is atheromatous plaque in major blood vessels, particularly the abdominal aorta.

 

 

Many organs and systems can be affected

These emboli can affect many organs and systems: eg, the kidneys (causing hypertension and acute renal failure), the muscles (causing myalgias), gastrointestinal organs (causing bleeding, abdominal pain, and bowel infarction), the lungs (causing acute respiratory distress syndrome), the eyes (causing Hollenhorst plaques in retinal arteries), and the central nervous system (causing stroke, confusion, and delirium). Cardiac or central nervous system involvement is associated with a high risk of death.

After angiography, clinical manifestations of cholesterol embolization have been reported in 0.06% to 1.4% of patients,2,3 although the finding of cholesterol emboli is more common in autopsy studies.4

Recognizing skin signs is the key

Cutaneous abnormalities are usually the earliest and often the only clinical manifestation of this syndrome. Findings on the lower limbs include blue toes, cutaneous nodules, and livedo reticularis, affecting the feet and legs and sometimes extending upward to the trunk. Other findings include digital infarcts, ulceration, gangrene, purpura, cyanosis, and splinter hemorrhages in the nail bed.

RTEmagicC_Arias-Santiago_SkinLesion_F2.gif.gif
Figure 2. Skin biopsy showed needle-shaped clefts within the lumen of blood vessels, ie, dissolved cholesterol crystals obstructing small arteries.

In our patient, microscopic study of skin biopsy specimens showed needle-shaped clefts within the lumen of blood vessels—ie, dissolved cholesterol crystals obstructing small arteries (Figure 2).

Biopsy studies of skin lesions are positive in a high percentage of cases, showing dissolved cholesterol crystals within the lumen of blood vessels, especially in the small to large arteries and arterioles of the deep dermis or subcutaneous fat. Deep biopsies and carefully examination are necessary, as emboli tend to be patchily distributed. Early recognition of cutaneous clinical findings is essential to establish the proper diagnosis and treatment.

The diagnostic triad of this disease includes blue toe syndrome, acute or subacute renal failure, and a temporal relation with an inciting event (particularly angiography). But despite these diagnostic criteria,2 the diagnosis is often based on a combination of signs and symptoms specific to end-organ damage and a systemic inflammatory response.3

Histologic confirmation is considered essential to the diagnosis of cholesterol crystal embolism, and as the skin is the most accessible site, skin biopsy provides the best sample for histologic diagnosis.5

Postprocedural embolism of a blood clot, vasculitis, and infective endocarditis are the most important differential diagnoses.6,7

Treatment is supportive, preventive

Treatment is mainly supportive with hemodynamic monitoring, nutritional and metabolic support, mechanical ventilation, and dialysis if necessary. The underlying atherosclerotic disease should be treated aggressively. Prevention of another episode involves modification of traditional risk factors such as serum cholesterol, diabetes, hypertension, and smoking. Additional vascular surgery procedures should be avoided, as they can induce new episodes.

A 64-year-old man with diabetes and hypertension presented with a 2-day history of sudden onset of acute pain and cyanosis on the sole of his right foot 4 days after undergoing cardiac catheterization and coronary angiography.

RTEmagicC_Arias-Santiago_SkinLesion_F1.gif.gif
Figure1. Macular violaceous connecting rings in a net-like pattern compatible with livedo reticularis on the foot.
The physical examination revealed macular, violaceous, connecting rings in a net-like pattern that blanched with pressure and disappeared when the foot was elevated, a presentation compatible with livedo reticularis (Figure 1). Laboratory testing (complete blood cell count, biochemistry panel, coagulation test, and C-reactive protein test) was notable only for eosinophilia.

A few days later, the patient returned with abdominal pain, diarrhea, and acute renal injury with urinary eosinophils (7% of the white blood cells in the urine) and proteinuria.

Q: Which is the most likely diagnosis?

  • Infective endocarditis
  • Pernio (chilblain)
  • Cholesterol crystal embolism
  • Cutaneous small-vessel vasculitis

A: Cholesterol crystal embolism is the correct diagnosis.

Infective endocarditis is an infection of the endocardium, but systemic signs may be present, including cutaneous lesions such as Osler nodes (painful papules on the tips of the fingers and toes) and Janeway lesions (painless macules on the palms and soles). Histologic staining of skin biopsy specimens often shows vasculitis, occasionally with a positive Gram stain. Severe renal injury is not common, and the timing of the acute illness and skin lesion fits better with an acute embolic phenomenon.

Pernio is a form of cold injury, localized in peripheral parts of the body and occurring after exposure to cold temperatures in damp conditions. It usually manifests bilaterally as painful erythematous or purple lesions on the acral areas of the hands and feet, nose, ears, and, rarely, the thighs and buttocks. Pernio most commonly affects women between 20 and 40 years of age. It can be idiopathic or associated with a systemic disease such as systemic lupus erythematosus or Sjögren syndrome.

Cutaneous small-vessel vasculitis is a heterogeneous group of disorders with inflammation and damage of the blood vessels; it may be limited to the skin or it may involve multiple systems. Palpable or nonpalpable purpura and ulceration are common clinical findings, and histologic study shows an inflammatory infiltrate of vessel walls, fibrinoid necrosis, thrombosis, and extravasation of red blood cells.

While this patient’s problems are not consistent with small-vessel vasculitis, the single skin lesion, the timing after the catheterization, and the urinary eosinophils are best explained by cholesterol embolization.

CHOLESTEROL CRYSTAL EMBOLISM

Cholesterol crystal embolism is commonly iatrogenic, a complication of mechanical damage to the arterial walls from vascular surgery or invasive percutaneous procedures. Material dislodged from atheromatous arterial plaques can occlude the small vessels of the feet, producing this syndrome.

The onset of the clinical disease is often delayed for days to weeks after an angiographic procedure.1 Spontaneous hemorrhage, disruption of plaque, or drug therapy with an anticoagulant or a fibrinolytic can precipitate embolization of cholesterol crystals. The source of the emboli is atheromatous plaque in major blood vessels, particularly the abdominal aorta.

 

 

Many organs and systems can be affected

These emboli can affect many organs and systems: eg, the kidneys (causing hypertension and acute renal failure), the muscles (causing myalgias), gastrointestinal organs (causing bleeding, abdominal pain, and bowel infarction), the lungs (causing acute respiratory distress syndrome), the eyes (causing Hollenhorst plaques in retinal arteries), and the central nervous system (causing stroke, confusion, and delirium). Cardiac or central nervous system involvement is associated with a high risk of death.

After angiography, clinical manifestations of cholesterol embolization have been reported in 0.06% to 1.4% of patients,2,3 although the finding of cholesterol emboli is more common in autopsy studies.4

Recognizing skin signs is the key

Cutaneous abnormalities are usually the earliest and often the only clinical manifestation of this syndrome. Findings on the lower limbs include blue toes, cutaneous nodules, and livedo reticularis, affecting the feet and legs and sometimes extending upward to the trunk. Other findings include digital infarcts, ulceration, gangrene, purpura, cyanosis, and splinter hemorrhages in the nail bed.

RTEmagicC_Arias-Santiago_SkinLesion_F2.gif.gif
Figure 2. Skin biopsy showed needle-shaped clefts within the lumen of blood vessels, ie, dissolved cholesterol crystals obstructing small arteries.

In our patient, microscopic study of skin biopsy specimens showed needle-shaped clefts within the lumen of blood vessels—ie, dissolved cholesterol crystals obstructing small arteries (Figure 2).

Biopsy studies of skin lesions are positive in a high percentage of cases, showing dissolved cholesterol crystals within the lumen of blood vessels, especially in the small to large arteries and arterioles of the deep dermis or subcutaneous fat. Deep biopsies and carefully examination are necessary, as emboli tend to be patchily distributed. Early recognition of cutaneous clinical findings is essential to establish the proper diagnosis and treatment.

The diagnostic triad of this disease includes blue toe syndrome, acute or subacute renal failure, and a temporal relation with an inciting event (particularly angiography). But despite these diagnostic criteria,2 the diagnosis is often based on a combination of signs and symptoms specific to end-organ damage and a systemic inflammatory response.3

Histologic confirmation is considered essential to the diagnosis of cholesterol crystal embolism, and as the skin is the most accessible site, skin biopsy provides the best sample for histologic diagnosis.5

Postprocedural embolism of a blood clot, vasculitis, and infective endocarditis are the most important differential diagnoses.6,7

Treatment is supportive, preventive

Treatment is mainly supportive with hemodynamic monitoring, nutritional and metabolic support, mechanical ventilation, and dialysis if necessary. The underlying atherosclerotic disease should be treated aggressively. Prevention of another episode involves modification of traditional risk factors such as serum cholesterol, diabetes, hypertension, and smoking. Additional vascular surgery procedures should be avoided, as they can induce new episodes.

References
  1. Donohue KG, Saap L, Falanga V. Cholesterol crystal embolization: an atherosclerotic disease with frequent and varied cutaneous manifestations. J Eur Acad Dermatol Venereol 2003; 17:504511.
  2. Fukumoto Y, Tsutsui H, Tsuchihashi M, Masumoto A, Takeshita A; Cholesterol Embolism Study (CHEST) Investigators. The incidence and risk factors of cholesterol embolization syndrome, a complication of cardiac catheterization: a prospective study. J Am Coll Cardiol 2003; 42:211216.
  3. Johnson LW, Esente P, Giambartolomei A, et al. Peripheral vascular complications of coronary angioplasty by the femoral and brachial techniques. Cathet Cardiovasc Diagn 1994; 31:165172.
  4. Kronzon I, Saric M. Cholesterol embolization syndrome. Circulation 2010; 122:631641.
  5. Jucgla A, Moreso F, Muniesa C, Moreno A, Vidaller A. Cholesterol embolism: still an unrecognized entity with a high mortality rate. J Am Acad Dermatol 2006; 55:786793.
  6. Maki T, Izumi C, Miyake M, et al. Cholesterol embolism after cardiac catheterization mimicking infective endocarditis. Intern Med 2005; 44:10601063.
  7. Arias-Santiago S, Aneiros-Fernández J, Girón-Prieto MS, Fernández-Pugnaire MA, Naranjo-Sintes R. Palpable purpura. Cleve Clin J Med 2010; 77:205206.
References
  1. Donohue KG, Saap L, Falanga V. Cholesterol crystal embolization: an atherosclerotic disease with frequent and varied cutaneous manifestations. J Eur Acad Dermatol Venereol 2003; 17:504511.
  2. Fukumoto Y, Tsutsui H, Tsuchihashi M, Masumoto A, Takeshita A; Cholesterol Embolism Study (CHEST) Investigators. The incidence and risk factors of cholesterol embolization syndrome, a complication of cardiac catheterization: a prospective study. J Am Coll Cardiol 2003; 42:211216.
  3. Johnson LW, Esente P, Giambartolomei A, et al. Peripheral vascular complications of coronary angioplasty by the femoral and brachial techniques. Cathet Cardiovasc Diagn 1994; 31:165172.
  4. Kronzon I, Saric M. Cholesterol embolization syndrome. Circulation 2010; 122:631641.
  5. Jucgla A, Moreso F, Muniesa C, Moreno A, Vidaller A. Cholesterol embolism: still an unrecognized entity with a high mortality rate. J Am Acad Dermatol 2006; 55:786793.
  6. Maki T, Izumi C, Miyake M, et al. Cholesterol embolism after cardiac catheterization mimicking infective endocarditis. Intern Med 2005; 44:10601063.
  7. Arias-Santiago S, Aneiros-Fernández J, Girón-Prieto MS, Fernández-Pugnaire MA, Naranjo-Sintes R. Palpable purpura. Cleve Clin J Med 2010; 77:205206.
Issue
Cleveland Clinic Journal of Medicine - 79(6)
Issue
Cleveland Clinic Journal of Medicine - 79(6)
Page Number
424-426
Page Number
424-426
Publications
Publications
Topics
Article Type
Display Headline
A skin lesion after cardiac catheterization
Display Headline
A skin lesion after cardiac catheterization
Sections
Disallow All Ads
Alternative CME
Article PDF Media
Image
Disable zoom
Off

A nodule on a woman’s face

Article Type
Changed
Mon, 10/02/2017 - 12:09
Display Headline
A nodule on a woman’s face

RTEmagicC_Arias-Santiago_NoduleontheFace_F1.gif.gif
Figure 1. A firm, pink nodule, 10 mm in diameter, with surrounding telangiectasias, at the site of a previous mosquito bite.
An otherwise healthy 54-year-old woman presented with a 3-month history of an asymptomatic lesion on the face, at the site of a previous mosquito bite. Physical examination revealed a firm, pink nodule 10 mm in diameter (Figure 1). Telangiectasias were visible, more clearly by dermoscopy, but other features of a basal cell carcinoma were absent. No other skin problem was noted, and no lymphadenopathy was detected.

RTEmagicC_f7f7525_283fig2.jpeg
Figure 2. In A, dense chronic inflammation (arrows) is noted in the upper and deeper dermis (hematoxylin-eosin, × 4). Immunohistochemical staining shows, in B, CD20+ B lymphocytes (arrow) with a central predominance (× 40); in C, a smaller number of CD3+ T lymphocytes (arrow) are present at the periphery of rudimentary germinal centers (× 40).
Histologic study of a biopsy specimen noted a dense dermal inflammatory infiltrate consisting of B lymphocytes (CD20+) with a smaller number of T lymphocytes (CD3+) arranged in several germinal centers, with an admixture of plasma cells and eosinophils (Figure 2). No clonal population was identified by gene-rearrangement studies. Borrelia burgdorferi infection was ruled out.

Q: Which is the most likely diagnosis?

  • Basal cell carcinoma
  • Squamous cell carcinoma
  • Lymphocytoma cutis
  • Amelanotic melanoma
  • Pyogenic granuloma

A: The correct answer is lymphocytoma cutis. The differential diagnosis of a pink papule on the face of a middle-aged person includes nonmelanoma skin cancer, lymphoma, lymphocytoma cutis, metastatic disease, certain infections, Jessner lymphocytic infiltrate, connective tissue disease, and some adnexal tumors. Histologic study is a useful diagnostic aid in this context.

Basal cell carcinoma is the most common cutaneous malignant neoplasm, and although these tumors rarely metastasize, they are capable of gross tissue destruction, particularly those lesions arising on the face. Clinically, this tumor presents as a shiny, pearly nodule with telangiectasias on the surface, as in our patient, but skin biopsy shows large basaloid lobules of varying shape and size forming a relatively circumscribed mass with a “palisade” around the rim of the lobule.

Squamous cell carcinoma manifests as shallow ulcers, often with a keratinous crust and elevated, indurate borders, but also as plaques or nodules. The clinical diagnosis should be confirmed with skin biopsy, which reveals atypical keratinocytes extending from the epidermis to the dermis with dyskeratosis, intercellular bridges, variable central keratinization, and horn pearl formation, depending on the differentiation of the tumor.

Amelanotic melanoma is nonpigmented and appears as a pink nodule mimicking basal cell carcinoma or squamous cell carcinoma. Histologic study is necessary for the diagnosis, and shows an atypical proliferation of melanocytic cells in the epidermis and dermis.

Pyogenic granuloma is a very common benign vascular lesion considered to be a hyperplastic process or a vascular neoplasm. The lesion typically presents as a red or bluish papule or polyp that bleeds easily, and a reddish homogeneous area surrounded by a white “collarette” is found in most cases. Histologic features of an early lesion resemble granulation tissue and include lobules of capillaries and venules that often radiate from larger, more central vessels.

LYMPHOCYTOMA CUTIS: KEY FEATURES

Lymphocytoma cutis (pseudolymphoma) is a benign reactive polyclonal and inflammatory disorder that most frequently includes B lymphocytes, with a smaller population of T lymphocytes. It infiltrates the skin and resembles rudimentary germinal follicles, as in the present case. The lesion usually presents as an asymptomatic red-brown or violet papule or nodule, 3 mm to 5 cm in diameter, most often on the face, chest, or upper extremities.1 The lesion may be solitary, as in our patient, but lesions may also be grouped or numerous and widespread. It is three times more common in women than in men. It may resolve spontaneously, but it may also recur.

In Europe, lymphocytoma cutis occurs most often in B burgdorferi infection after a tick bite. Lymphocytoma cutis occurs in 1.3% of cases of B burgdorferi infection,2 although other infectious, physical, or chemical agents may produce the same reaction pattern. Tattooing (particularly red areas), acupuncture, vaccination, arthropod reactions, hyposensitization antigen reaction, and ingestion of drug have been implicated in this form of lymphoid hyperplasia.3,4

DIAGNOSTIC CHALLENGES

Lymphocytoma cutis can be challenging to diagnose, and although it can be suspected clinically, incisional biopsy is usually necessary in order to differentiate it from cutaneous B lymphoma.5

The infiltrate is predominantly nodular (> 90%) and located in the upper and mid dermis (“top heavy”) in lymphocytoma cutis, whereas it can be nodular or diffuse in cutaneous B lymphoma, with sharply demarcated borders that are convex rather than concave. Lymphoid follicles with germinal centers are sometimes present, and the interfollicular cellular population is polymorphic in lymphocytoma cutis (lymphocytes, plasma cells, histiocytes, eosinophils). In lymphocytoma cutis, cells express the phenotype of mature B lymphocytes (CD20, CD79a) and show regular and sharply demarcated networks of CD21+ follicular dendritic cells, whereas in cutaneous B lymphoma these networks are irregular. Light chains are usually polyclonal, although monoclonal populations of B cell in cases of cutaneous lymphocytoma cutis have been described. Extracutaneous involvement is possible in cutaneous B lymphoma but is usually absent in lymphocytoma cutis.

Lymphocytoma cutis typically involutes over a period of months, even with no treatment, as it did in our patient. Otherwise, there are different therapeutic options, including intralesional and topical corticosteroids, surgery, and cryosurgery.6 Photodynamic therapy with delta-aminolevulinic acid is an effective and safe modality for the treatment of lymphocytoma cutis and may be cosmetically beneficial.7

References
  1. Ploysangam T, Breneman DL, Mutasim DF. Cutaneous pseudolymphomas. J Am Acad Dermatol 1998; 38:877895.
  2. Albrecht S, Hofstadter S, Artsob H, Chaban O, From L. Lymphadenosis benigna cutis resulting from Borrelia infection (Borrelia lymphocytoma). J Am Acad Dermatol 1991; 24:621625.
  3. Peretz E, Grunwald MH, Cagnano E, Halevy S. Follicular B-cell pseudolymphoma. Australas J Dermatol 2000; 41:4849.
  4. Hermes B, Haas N, Grabbe J, Czarnetzki BM. Foreign-body granuloma and IgE-pseudolymphoma after multiple bee stings. Br J Dermatol 1994; 130:780784.
  5. Kerl H, Fink-Puches R, Cerroni L. Diagnostic criteria of primary cutaneous B-cell lymphomas and pseudolymphomas. Keio J Med 2001; 50:269273.
  6. Kuflik AS, Schwartz RA. Lymphocytoma cutis: a series of five patients successfully treated with cryosurgery. J Am Acad Dermatol 1992; 26:449452.
  7. Takeda H, Kaneko T, Harada K, Matsuzaki Y, Nakano H, Hanada K. Successful treatment of lymphadenosis benigna cutis with topical photodynamic therapy with delta-aminolevulinic acid. Dermatology 2005; 211:264266.
Article PDF
Author and Disclosure Information

Salvador Arias-Santiago, MD
Department of Dermatology, San Cecilio University Hospital, Granada, Spain

José Aneiros-Fernández, MD
Department of Pathology, San Cecilio University Hospital, Granada, Spain

Antonio Cutando, PhD
Department of Pathology, San Cecilio University Hospital, Granada, Spain

Agustín Buendía-Eisman, PhD
Department of Dermatology, San Cecilio University Hospital, Granada, Spain

Ramón Naranjo-Sintes, PhD
Department of Dermatology, San Cecilio University Hospital, Granada, Spain

Antonio Campos, PhD
Department of Histology, School of Medicine, Granada, Spain

Miguel Alaminos-Mingorance
Department of Histology, School of Medicine, Granada, Spain

Address: Salvador Arias-Santiago, MD, Department of Dermatology, San Cecilio University Hospital, Av Dr. Olóriz 16, Granada 18012, Spain; e-mail salvadorarias@hotmail.es

Issue
Cleveland Clinic Journal of Medicine - 79(4)
Publications
Topics
Page Number
282-284
Sections
Author and Disclosure Information

Salvador Arias-Santiago, MD
Department of Dermatology, San Cecilio University Hospital, Granada, Spain

José Aneiros-Fernández, MD
Department of Pathology, San Cecilio University Hospital, Granada, Spain

Antonio Cutando, PhD
Department of Pathology, San Cecilio University Hospital, Granada, Spain

Agustín Buendía-Eisman, PhD
Department of Dermatology, San Cecilio University Hospital, Granada, Spain

Ramón Naranjo-Sintes, PhD
Department of Dermatology, San Cecilio University Hospital, Granada, Spain

Antonio Campos, PhD
Department of Histology, School of Medicine, Granada, Spain

Miguel Alaminos-Mingorance
Department of Histology, School of Medicine, Granada, Spain

Address: Salvador Arias-Santiago, MD, Department of Dermatology, San Cecilio University Hospital, Av Dr. Olóriz 16, Granada 18012, Spain; e-mail salvadorarias@hotmail.es

Author and Disclosure Information

Salvador Arias-Santiago, MD
Department of Dermatology, San Cecilio University Hospital, Granada, Spain

José Aneiros-Fernández, MD
Department of Pathology, San Cecilio University Hospital, Granada, Spain

Antonio Cutando, PhD
Department of Pathology, San Cecilio University Hospital, Granada, Spain

Agustín Buendía-Eisman, PhD
Department of Dermatology, San Cecilio University Hospital, Granada, Spain

Ramón Naranjo-Sintes, PhD
Department of Dermatology, San Cecilio University Hospital, Granada, Spain

Antonio Campos, PhD
Department of Histology, School of Medicine, Granada, Spain

Miguel Alaminos-Mingorance
Department of Histology, School of Medicine, Granada, Spain

Address: Salvador Arias-Santiago, MD, Department of Dermatology, San Cecilio University Hospital, Av Dr. Olóriz 16, Granada 18012, Spain; e-mail salvadorarias@hotmail.es

Article PDF
Article PDF

RTEmagicC_Arias-Santiago_NoduleontheFace_F1.gif.gif
Figure 1. A firm, pink nodule, 10 mm in diameter, with surrounding telangiectasias, at the site of a previous mosquito bite.
An otherwise healthy 54-year-old woman presented with a 3-month history of an asymptomatic lesion on the face, at the site of a previous mosquito bite. Physical examination revealed a firm, pink nodule 10 mm in diameter (Figure 1). Telangiectasias were visible, more clearly by dermoscopy, but other features of a basal cell carcinoma were absent. No other skin problem was noted, and no lymphadenopathy was detected.

RTEmagicC_f7f7525_283fig2.jpeg
Figure 2. In A, dense chronic inflammation (arrows) is noted in the upper and deeper dermis (hematoxylin-eosin, × 4). Immunohistochemical staining shows, in B, CD20+ B lymphocytes (arrow) with a central predominance (× 40); in C, a smaller number of CD3+ T lymphocytes (arrow) are present at the periphery of rudimentary germinal centers (× 40).
Histologic study of a biopsy specimen noted a dense dermal inflammatory infiltrate consisting of B lymphocytes (CD20+) with a smaller number of T lymphocytes (CD3+) arranged in several germinal centers, with an admixture of plasma cells and eosinophils (Figure 2). No clonal population was identified by gene-rearrangement studies. Borrelia burgdorferi infection was ruled out.

Q: Which is the most likely diagnosis?

  • Basal cell carcinoma
  • Squamous cell carcinoma
  • Lymphocytoma cutis
  • Amelanotic melanoma
  • Pyogenic granuloma

A: The correct answer is lymphocytoma cutis. The differential diagnosis of a pink papule on the face of a middle-aged person includes nonmelanoma skin cancer, lymphoma, lymphocytoma cutis, metastatic disease, certain infections, Jessner lymphocytic infiltrate, connective tissue disease, and some adnexal tumors. Histologic study is a useful diagnostic aid in this context.

Basal cell carcinoma is the most common cutaneous malignant neoplasm, and although these tumors rarely metastasize, they are capable of gross tissue destruction, particularly those lesions arising on the face. Clinically, this tumor presents as a shiny, pearly nodule with telangiectasias on the surface, as in our patient, but skin biopsy shows large basaloid lobules of varying shape and size forming a relatively circumscribed mass with a “palisade” around the rim of the lobule.

Squamous cell carcinoma manifests as shallow ulcers, often with a keratinous crust and elevated, indurate borders, but also as plaques or nodules. The clinical diagnosis should be confirmed with skin biopsy, which reveals atypical keratinocytes extending from the epidermis to the dermis with dyskeratosis, intercellular bridges, variable central keratinization, and horn pearl formation, depending on the differentiation of the tumor.

Amelanotic melanoma is nonpigmented and appears as a pink nodule mimicking basal cell carcinoma or squamous cell carcinoma. Histologic study is necessary for the diagnosis, and shows an atypical proliferation of melanocytic cells in the epidermis and dermis.

Pyogenic granuloma is a very common benign vascular lesion considered to be a hyperplastic process or a vascular neoplasm. The lesion typically presents as a red or bluish papule or polyp that bleeds easily, and a reddish homogeneous area surrounded by a white “collarette” is found in most cases. Histologic features of an early lesion resemble granulation tissue and include lobules of capillaries and venules that often radiate from larger, more central vessels.

LYMPHOCYTOMA CUTIS: KEY FEATURES

Lymphocytoma cutis (pseudolymphoma) is a benign reactive polyclonal and inflammatory disorder that most frequently includes B lymphocytes, with a smaller population of T lymphocytes. It infiltrates the skin and resembles rudimentary germinal follicles, as in the present case. The lesion usually presents as an asymptomatic red-brown or violet papule or nodule, 3 mm to 5 cm in diameter, most often on the face, chest, or upper extremities.1 The lesion may be solitary, as in our patient, but lesions may also be grouped or numerous and widespread. It is three times more common in women than in men. It may resolve spontaneously, but it may also recur.

In Europe, lymphocytoma cutis occurs most often in B burgdorferi infection after a tick bite. Lymphocytoma cutis occurs in 1.3% of cases of B burgdorferi infection,2 although other infectious, physical, or chemical agents may produce the same reaction pattern. Tattooing (particularly red areas), acupuncture, vaccination, arthropod reactions, hyposensitization antigen reaction, and ingestion of drug have been implicated in this form of lymphoid hyperplasia.3,4

DIAGNOSTIC CHALLENGES

Lymphocytoma cutis can be challenging to diagnose, and although it can be suspected clinically, incisional biopsy is usually necessary in order to differentiate it from cutaneous B lymphoma.5

The infiltrate is predominantly nodular (> 90%) and located in the upper and mid dermis (“top heavy”) in lymphocytoma cutis, whereas it can be nodular or diffuse in cutaneous B lymphoma, with sharply demarcated borders that are convex rather than concave. Lymphoid follicles with germinal centers are sometimes present, and the interfollicular cellular population is polymorphic in lymphocytoma cutis (lymphocytes, plasma cells, histiocytes, eosinophils). In lymphocytoma cutis, cells express the phenotype of mature B lymphocytes (CD20, CD79a) and show regular and sharply demarcated networks of CD21+ follicular dendritic cells, whereas in cutaneous B lymphoma these networks are irregular. Light chains are usually polyclonal, although monoclonal populations of B cell in cases of cutaneous lymphocytoma cutis have been described. Extracutaneous involvement is possible in cutaneous B lymphoma but is usually absent in lymphocytoma cutis.

Lymphocytoma cutis typically involutes over a period of months, even with no treatment, as it did in our patient. Otherwise, there are different therapeutic options, including intralesional and topical corticosteroids, surgery, and cryosurgery.6 Photodynamic therapy with delta-aminolevulinic acid is an effective and safe modality for the treatment of lymphocytoma cutis and may be cosmetically beneficial.7

RTEmagicC_Arias-Santiago_NoduleontheFace_F1.gif.gif
Figure 1. A firm, pink nodule, 10 mm in diameter, with surrounding telangiectasias, at the site of a previous mosquito bite.
An otherwise healthy 54-year-old woman presented with a 3-month history of an asymptomatic lesion on the face, at the site of a previous mosquito bite. Physical examination revealed a firm, pink nodule 10 mm in diameter (Figure 1). Telangiectasias were visible, more clearly by dermoscopy, but other features of a basal cell carcinoma were absent. No other skin problem was noted, and no lymphadenopathy was detected.

RTEmagicC_f7f7525_283fig2.jpeg
Figure 2. In A, dense chronic inflammation (arrows) is noted in the upper and deeper dermis (hematoxylin-eosin, × 4). Immunohistochemical staining shows, in B, CD20+ B lymphocytes (arrow) with a central predominance (× 40); in C, a smaller number of CD3+ T lymphocytes (arrow) are present at the periphery of rudimentary germinal centers (× 40).
Histologic study of a biopsy specimen noted a dense dermal inflammatory infiltrate consisting of B lymphocytes (CD20+) with a smaller number of T lymphocytes (CD3+) arranged in several germinal centers, with an admixture of plasma cells and eosinophils (Figure 2). No clonal population was identified by gene-rearrangement studies. Borrelia burgdorferi infection was ruled out.

Q: Which is the most likely diagnosis?

  • Basal cell carcinoma
  • Squamous cell carcinoma
  • Lymphocytoma cutis
  • Amelanotic melanoma
  • Pyogenic granuloma

A: The correct answer is lymphocytoma cutis. The differential diagnosis of a pink papule on the face of a middle-aged person includes nonmelanoma skin cancer, lymphoma, lymphocytoma cutis, metastatic disease, certain infections, Jessner lymphocytic infiltrate, connective tissue disease, and some adnexal tumors. Histologic study is a useful diagnostic aid in this context.

Basal cell carcinoma is the most common cutaneous malignant neoplasm, and although these tumors rarely metastasize, they are capable of gross tissue destruction, particularly those lesions arising on the face. Clinically, this tumor presents as a shiny, pearly nodule with telangiectasias on the surface, as in our patient, but skin biopsy shows large basaloid lobules of varying shape and size forming a relatively circumscribed mass with a “palisade” around the rim of the lobule.

Squamous cell carcinoma manifests as shallow ulcers, often with a keratinous crust and elevated, indurate borders, but also as plaques or nodules. The clinical diagnosis should be confirmed with skin biopsy, which reveals atypical keratinocytes extending from the epidermis to the dermis with dyskeratosis, intercellular bridges, variable central keratinization, and horn pearl formation, depending on the differentiation of the tumor.

Amelanotic melanoma is nonpigmented and appears as a pink nodule mimicking basal cell carcinoma or squamous cell carcinoma. Histologic study is necessary for the diagnosis, and shows an atypical proliferation of melanocytic cells in the epidermis and dermis.

Pyogenic granuloma is a very common benign vascular lesion considered to be a hyperplastic process or a vascular neoplasm. The lesion typically presents as a red or bluish papule or polyp that bleeds easily, and a reddish homogeneous area surrounded by a white “collarette” is found in most cases. Histologic features of an early lesion resemble granulation tissue and include lobules of capillaries and venules that often radiate from larger, more central vessels.

LYMPHOCYTOMA CUTIS: KEY FEATURES

Lymphocytoma cutis (pseudolymphoma) is a benign reactive polyclonal and inflammatory disorder that most frequently includes B lymphocytes, with a smaller population of T lymphocytes. It infiltrates the skin and resembles rudimentary germinal follicles, as in the present case. The lesion usually presents as an asymptomatic red-brown or violet papule or nodule, 3 mm to 5 cm in diameter, most often on the face, chest, or upper extremities.1 The lesion may be solitary, as in our patient, but lesions may also be grouped or numerous and widespread. It is three times more common in women than in men. It may resolve spontaneously, but it may also recur.

In Europe, lymphocytoma cutis occurs most often in B burgdorferi infection after a tick bite. Lymphocytoma cutis occurs in 1.3% of cases of B burgdorferi infection,2 although other infectious, physical, or chemical agents may produce the same reaction pattern. Tattooing (particularly red areas), acupuncture, vaccination, arthropod reactions, hyposensitization antigen reaction, and ingestion of drug have been implicated in this form of lymphoid hyperplasia.3,4

DIAGNOSTIC CHALLENGES

Lymphocytoma cutis can be challenging to diagnose, and although it can be suspected clinically, incisional biopsy is usually necessary in order to differentiate it from cutaneous B lymphoma.5

The infiltrate is predominantly nodular (> 90%) and located in the upper and mid dermis (“top heavy”) in lymphocytoma cutis, whereas it can be nodular or diffuse in cutaneous B lymphoma, with sharply demarcated borders that are convex rather than concave. Lymphoid follicles with germinal centers are sometimes present, and the interfollicular cellular population is polymorphic in lymphocytoma cutis (lymphocytes, plasma cells, histiocytes, eosinophils). In lymphocytoma cutis, cells express the phenotype of mature B lymphocytes (CD20, CD79a) and show regular and sharply demarcated networks of CD21+ follicular dendritic cells, whereas in cutaneous B lymphoma these networks are irregular. Light chains are usually polyclonal, although monoclonal populations of B cell in cases of cutaneous lymphocytoma cutis have been described. Extracutaneous involvement is possible in cutaneous B lymphoma but is usually absent in lymphocytoma cutis.

Lymphocytoma cutis typically involutes over a period of months, even with no treatment, as it did in our patient. Otherwise, there are different therapeutic options, including intralesional and topical corticosteroids, surgery, and cryosurgery.6 Photodynamic therapy with delta-aminolevulinic acid is an effective and safe modality for the treatment of lymphocytoma cutis and may be cosmetically beneficial.7

References
  1. Ploysangam T, Breneman DL, Mutasim DF. Cutaneous pseudolymphomas. J Am Acad Dermatol 1998; 38:877895.
  2. Albrecht S, Hofstadter S, Artsob H, Chaban O, From L. Lymphadenosis benigna cutis resulting from Borrelia infection (Borrelia lymphocytoma). J Am Acad Dermatol 1991; 24:621625.
  3. Peretz E, Grunwald MH, Cagnano E, Halevy S. Follicular B-cell pseudolymphoma. Australas J Dermatol 2000; 41:4849.
  4. Hermes B, Haas N, Grabbe J, Czarnetzki BM. Foreign-body granuloma and IgE-pseudolymphoma after multiple bee stings. Br J Dermatol 1994; 130:780784.
  5. Kerl H, Fink-Puches R, Cerroni L. Diagnostic criteria of primary cutaneous B-cell lymphomas and pseudolymphomas. Keio J Med 2001; 50:269273.
  6. Kuflik AS, Schwartz RA. Lymphocytoma cutis: a series of five patients successfully treated with cryosurgery. J Am Acad Dermatol 1992; 26:449452.
  7. Takeda H, Kaneko T, Harada K, Matsuzaki Y, Nakano H, Hanada K. Successful treatment of lymphadenosis benigna cutis with topical photodynamic therapy with delta-aminolevulinic acid. Dermatology 2005; 211:264266.
References
  1. Ploysangam T, Breneman DL, Mutasim DF. Cutaneous pseudolymphomas. J Am Acad Dermatol 1998; 38:877895.
  2. Albrecht S, Hofstadter S, Artsob H, Chaban O, From L. Lymphadenosis benigna cutis resulting from Borrelia infection (Borrelia lymphocytoma). J Am Acad Dermatol 1991; 24:621625.
  3. Peretz E, Grunwald MH, Cagnano E, Halevy S. Follicular B-cell pseudolymphoma. Australas J Dermatol 2000; 41:4849.
  4. Hermes B, Haas N, Grabbe J, Czarnetzki BM. Foreign-body granuloma and IgE-pseudolymphoma after multiple bee stings. Br J Dermatol 1994; 130:780784.
  5. Kerl H, Fink-Puches R, Cerroni L. Diagnostic criteria of primary cutaneous B-cell lymphomas and pseudolymphomas. Keio J Med 2001; 50:269273.
  6. Kuflik AS, Schwartz RA. Lymphocytoma cutis: a series of five patients successfully treated with cryosurgery. J Am Acad Dermatol 1992; 26:449452.
  7. Takeda H, Kaneko T, Harada K, Matsuzaki Y, Nakano H, Hanada K. Successful treatment of lymphadenosis benigna cutis with topical photodynamic therapy with delta-aminolevulinic acid. Dermatology 2005; 211:264266.
Issue
Cleveland Clinic Journal of Medicine - 79(4)
Issue
Cleveland Clinic Journal of Medicine - 79(4)
Page Number
282-284
Page Number
282-284
Publications
Publications
Topics
Article Type
Display Headline
A nodule on a woman’s face
Display Headline
A nodule on a woman’s face
Sections
Disallow All Ads
Alternative CME
Article PDF Media
Image
Disable zoom
Off

An erythematous plaque on the nose

Article Type
Changed
Mon, 11/13/2017 - 08:47
Display Headline
An erythematous plaque on the nose

A 38-year-old woman presented with a pruriginous and erythematous lesion on her nose that appeared during periods of cold weather. She said she is completely asymptomatic during the summer months.

RTEmagicC_Arias-Santiago_ChilblainLupus_F1.gif.gif
Figure 1. The acrocyanotic lesions were covered with scales.
A physical examination revealed acrocyanotic lesions on the nose that were covered with scales (Figure 1). Laboratory testing showed increased cholesterol levels, a positive antinuclear antibody titer (1:160 or higher is positive), and a positive anti-Ro/SS-A antibody titer (1:80 or higher is positive). Tests for cryoglobulin, cold agglutinins, anti-double-stranded DNA antibody, anti-extractable nuclear antigens, C3 and C4 complement proteins, and anticardiolipin antibody were normal or negative.

RTEmagicC_52baa8f_729fig2.jpeg
Figure 2. On the left, superficial, interstitial, and deep perivascular and perifollicular dense infiltrate of lymphocytes is seen (arrows) (hematoxylin-eosin, × 4). On the right, hydropic degeneration of the basal cell layer is seen (arrow) (hematoxylin-eosin, × 40).
Histologic examination revealed degeneration of the basal layer of the dermis, with periadnexal and perivascular inflammatory infiltrates (Figure 2). On immunofluorescence testing, linear deposits of immunoglobulin M were noted at the dermoepidermal junction.

Q: What is the most likely diagnosis?

  • Lupus pernio
  • Rosacea
  • Seborrheic dermatitis
  • Chilblain lupus erythematosus
  • Lupus vulgaris

A: The diagnosis is chilblain lupus erythematosus.

The differential diagnosis of an erythematous lesion on the nose of a middle-aged woman also includes rosacea, lupus pernio, lupus vulgaris, and seborrheic dermatitis. Some of these lesions are exacerbated by cold. Usually, the diagnosis is based on clinical findings, but in some cases histologic features on biopsy study confirm the diagnosis.

Lesions of lupus pernio (sarcoidosis) remain unaltered with changes in temperature, and biopsy study usually shows granulomas without caseous necrosis with little inflammatory infiltrate at the periphery.

Rosacea usually gets worse with heat and with alcohol consumption, although it can be exacerbated by cold. Biopsy study shows a nonspecific perivascular and perifollicular lymphohistiocytic infiltrate accompanied occasionally by multinucleated cells.

Seborrheic dermatitis is a papulosquamous disorder characterized by greasy scaling over inflamed skin on the scalp, face, and trunk. Disease activity is increased in winter and spring, with remissions commonly occurring in summer. The histologic features of seborrheic dermatitis are nonspecific; in this case, the histologic features were compatible with chilblain lupus without changes of seborrheic dermatitis.

Lupus vulgaris is a chronic form of cutaneous tuberculosis characterized by redbrown papules with central atrophy. The nose and ears are usually affected. Histologically, granulomatous tubercles with epithelioid cells and caseation necrosis are usually found.

CHILBLAIN LUPUS ERYTHEMATOSUS

Pernio, or chilblain, is a localized inflammatory lesion of the skin resulting from an abnormal response to cold.1 The cutaneous lesions of chilblain may be classified as idiopathic, autoimmune-related (as in systemic lupus erythematosus, subacute cutaneous lupus), and induced by drugs such as terbinafine (Lamisil)2 or infliximab (Remicade).,3

Chilblain lupus is a rare form of cutaneous lupus erythematosus and should not be confused with lupus pernio, which is a misleading name used for a type of cutaneous sarcoidosis.4

Chilblain lupus is characterized by reddish-purple plaques in acral areas (more often the hands and feet, but also the nose and ears) that are induced by exposure to cold—unlike other lesions of lupus erythematosus, which worsen with exposure to sunlight. The main difference from the cutaneous variety of sarcoidosis (lupus pernio) is the histopathologic appearance. In patients with chilblain lupus, epidermal atrophy, perivascular and periadnexal inflammatory infiltrates, and degeneration of the basal layer are found, whereas in lupus pernio (sarcoidosis), we observe granulomas without caseous necrosis, but with few inflammatory infiltrates on the periphery.

PROPOSED DIAGNOSTIC CRITERIA

Su et al5 have proposed diagnostic criteria for chilblain lupus. Their two major criteria are skin lesions in acral locations induced by exposure to cold or a drop in temperature, and evidence of lupus erythematosus in the skin lesions by histopathologic examination or immunofluorescence study. Both of these criteria must be met, plus one of three minor criteria: the coexistence of systemic lupus erythematosus or of skin lesions of discoid lupus erythematosus; response to lupus therapy; and negative results of testing for cryoglobulin and cold agglutinins.

CHILBLAIN LUPUS VS SYSTEMIC LUPUS

Chilblain lupus is an uncommon manifestation of systemic lupus erythematosus, and it is reported to occur in about 20% of patients with that condition.6 Often, the onset of chilblain lupus precedes the systemic disease. Patients with systemic lupus erythematosus and chilblain lupus do not usually present with renal disease, mucosal lesions, or central nervous system involvement. However, Raynaud phenomenon and photosensitivity have been reported to be more frequently associated with chilblain lupus.7

A disorder of peripheral circulation could be involved in the pathogenesis of chilblain lupus, and the association with Raynaud phenomenon, livedo reticularis, antiphospholipid syndrome, and changes in nailfold capillaries supports this hypothesis. Antinuclear antibody and anti-Ro/SS-A antibody are commonly detected in the serum of patients with chilblain lupus, and anti-Ro/SS-A antibody seems to be a major serologic marker of chilblain lupus in patients with systemic lupus erythematosus.7

TREATMENT

Protection from cold by physical measures is very important, as well as the use of topical or oral antibiotics if the lesions are infected. In severe cases unresponsive to topical corticosteroids, a calcium channel blocker is a good therapeutic option; antimalarials, commonly used in the treatment of lupus erythematosus, can also have a positive effect in patients with chilblain lupus.

CASE CONCLUDED

Our patient was advised to protect herself from the cold. Topical corticosteroids and oral hydroxychloroquine (200 mg/day) were prescribed, and they produced a good response. In severe cases, oral corticosteroids, etretinate (Tegison), mycophenolate (CellCept), or thalidomide (Thalomid) may be used.8

References
  1. Simon TD, Soep JB, Hollister JR. Pernio in pediatrics. Pediatrics 2005; 116:e472e475.
  2. Bonsmann G, Schiller M, Luger TA, Ständer S. Terbinafine-induced subacute cutaneous lupus erythematosus. J Am Acad Dermatol 2001; 44:925931.
  3. Richez C, Dumoulin C, Schaeverbeke T. Infliximab induced chilblain lupus in a patient with rheumatoid arthritis. J Rheumatol 2005; 32:760761.
  4. Arias-Santiago SA, Girón-Prieto MS, Callejas-Rubio JL, Fernández-Pugnaire MA, Ortego-Centeno N. Lupus pernio or chilblain lupus?: two different entities. Chest 2009; 136:946947.
  5. Su WP, Perniciaro C, Rogers RS, White JW. Chilblain lupus erythematosus (lupus pernio): clinical review of the Mayo Clinic experience and proposal of diagnostic criteria. Cutis 1994; 54:395399.
  6. Yell JA, Mbuagbaw J, Burge SM. Cutaneous manifestations of systemic lupus erythematosus. Br J Dermatol 1996; 135:355362.
  7. Franceschini F, Calzavara-Pinton P, Quinzanini M, et al. Chilblain lupus erythematosus is associated with antibodies to SSA/Ro. Lupus 1999; 8:215219.
  8. Bouaziz JD, Barete S, Le Pelletier F, Amoura Z, Piette JC, Francès C. Cutaneous lesions of the digits in systemic lupus erythematosus: 50 cases. Lupus 2007; 16:163167.
Article PDF
Author and Disclosure Information

Salvador Arias-Santiago, MD, PhD
Department of Dermatology, San Cecilio University Hospital, Granada, Spain

María Isabel Soriano-Hernández, MD
Department of Dermatology, San Cecilio University Hospital, Granada, Spain

José Aneiros-Fernández, MD
Department of Pathology, San Cecilio University Hospital, Granada, Spain

Pilar Burkhardt-Pérez, PhD
Department of Dermatology, San Cecilio University Hospital, Granada, Spain

Agustín Buendía-Eisman, PhD
Department of Dermatology, San Cecilio University Hospital, Granada, Spain

Ramón Naranjo-Sintes, PhD
Department of Dermatology, San Cecilio University Hospital, Granada, Spain

Miguel Alaminos-Mingorance, PhD
Department of Histology, School of Medicine, Granada, Spain

Address: Salvador Arias-Santiago, MD, Department of Dermatology, San Cecilio University Hospital, Av Dr. Olóriz 16, Granada 18012, Spain; e-mail salvadorarias@hotmail.es

Issue
Cleveland Clinic Journal of Medicine - 78(11)
Publications
Topics
Page Number
728-732
Sections
Author and Disclosure Information

Salvador Arias-Santiago, MD, PhD
Department of Dermatology, San Cecilio University Hospital, Granada, Spain

María Isabel Soriano-Hernández, MD
Department of Dermatology, San Cecilio University Hospital, Granada, Spain

José Aneiros-Fernández, MD
Department of Pathology, San Cecilio University Hospital, Granada, Spain

Pilar Burkhardt-Pérez, PhD
Department of Dermatology, San Cecilio University Hospital, Granada, Spain

Agustín Buendía-Eisman, PhD
Department of Dermatology, San Cecilio University Hospital, Granada, Spain

Ramón Naranjo-Sintes, PhD
Department of Dermatology, San Cecilio University Hospital, Granada, Spain

Miguel Alaminos-Mingorance, PhD
Department of Histology, School of Medicine, Granada, Spain

Address: Salvador Arias-Santiago, MD, Department of Dermatology, San Cecilio University Hospital, Av Dr. Olóriz 16, Granada 18012, Spain; e-mail salvadorarias@hotmail.es

Author and Disclosure Information

Salvador Arias-Santiago, MD, PhD
Department of Dermatology, San Cecilio University Hospital, Granada, Spain

María Isabel Soriano-Hernández, MD
Department of Dermatology, San Cecilio University Hospital, Granada, Spain

José Aneiros-Fernández, MD
Department of Pathology, San Cecilio University Hospital, Granada, Spain

Pilar Burkhardt-Pérez, PhD
Department of Dermatology, San Cecilio University Hospital, Granada, Spain

Agustín Buendía-Eisman, PhD
Department of Dermatology, San Cecilio University Hospital, Granada, Spain

Ramón Naranjo-Sintes, PhD
Department of Dermatology, San Cecilio University Hospital, Granada, Spain

Miguel Alaminos-Mingorance, PhD
Department of Histology, School of Medicine, Granada, Spain

Address: Salvador Arias-Santiago, MD, Department of Dermatology, San Cecilio University Hospital, Av Dr. Olóriz 16, Granada 18012, Spain; e-mail salvadorarias@hotmail.es

Article PDF
Article PDF

A 38-year-old woman presented with a pruriginous and erythematous lesion on her nose that appeared during periods of cold weather. She said she is completely asymptomatic during the summer months.

RTEmagicC_Arias-Santiago_ChilblainLupus_F1.gif.gif
Figure 1. The acrocyanotic lesions were covered with scales.
A physical examination revealed acrocyanotic lesions on the nose that were covered with scales (Figure 1). Laboratory testing showed increased cholesterol levels, a positive antinuclear antibody titer (1:160 or higher is positive), and a positive anti-Ro/SS-A antibody titer (1:80 or higher is positive). Tests for cryoglobulin, cold agglutinins, anti-double-stranded DNA antibody, anti-extractable nuclear antigens, C3 and C4 complement proteins, and anticardiolipin antibody were normal or negative.

RTEmagicC_52baa8f_729fig2.jpeg
Figure 2. On the left, superficial, interstitial, and deep perivascular and perifollicular dense infiltrate of lymphocytes is seen (arrows) (hematoxylin-eosin, × 4). On the right, hydropic degeneration of the basal cell layer is seen (arrow) (hematoxylin-eosin, × 40).
Histologic examination revealed degeneration of the basal layer of the dermis, with periadnexal and perivascular inflammatory infiltrates (Figure 2). On immunofluorescence testing, linear deposits of immunoglobulin M were noted at the dermoepidermal junction.

Q: What is the most likely diagnosis?

  • Lupus pernio
  • Rosacea
  • Seborrheic dermatitis
  • Chilblain lupus erythematosus
  • Lupus vulgaris

A: The diagnosis is chilblain lupus erythematosus.

The differential diagnosis of an erythematous lesion on the nose of a middle-aged woman also includes rosacea, lupus pernio, lupus vulgaris, and seborrheic dermatitis. Some of these lesions are exacerbated by cold. Usually, the diagnosis is based on clinical findings, but in some cases histologic features on biopsy study confirm the diagnosis.

Lesions of lupus pernio (sarcoidosis) remain unaltered with changes in temperature, and biopsy study usually shows granulomas without caseous necrosis with little inflammatory infiltrate at the periphery.

Rosacea usually gets worse with heat and with alcohol consumption, although it can be exacerbated by cold. Biopsy study shows a nonspecific perivascular and perifollicular lymphohistiocytic infiltrate accompanied occasionally by multinucleated cells.

Seborrheic dermatitis is a papulosquamous disorder characterized by greasy scaling over inflamed skin on the scalp, face, and trunk. Disease activity is increased in winter and spring, with remissions commonly occurring in summer. The histologic features of seborrheic dermatitis are nonspecific; in this case, the histologic features were compatible with chilblain lupus without changes of seborrheic dermatitis.

Lupus vulgaris is a chronic form of cutaneous tuberculosis characterized by redbrown papules with central atrophy. The nose and ears are usually affected. Histologically, granulomatous tubercles with epithelioid cells and caseation necrosis are usually found.

CHILBLAIN LUPUS ERYTHEMATOSUS

Pernio, or chilblain, is a localized inflammatory lesion of the skin resulting from an abnormal response to cold.1 The cutaneous lesions of chilblain may be classified as idiopathic, autoimmune-related (as in systemic lupus erythematosus, subacute cutaneous lupus), and induced by drugs such as terbinafine (Lamisil)2 or infliximab (Remicade).,3

Chilblain lupus is a rare form of cutaneous lupus erythematosus and should not be confused with lupus pernio, which is a misleading name used for a type of cutaneous sarcoidosis.4

Chilblain lupus is characterized by reddish-purple plaques in acral areas (more often the hands and feet, but also the nose and ears) that are induced by exposure to cold—unlike other lesions of lupus erythematosus, which worsen with exposure to sunlight. The main difference from the cutaneous variety of sarcoidosis (lupus pernio) is the histopathologic appearance. In patients with chilblain lupus, epidermal atrophy, perivascular and periadnexal inflammatory infiltrates, and degeneration of the basal layer are found, whereas in lupus pernio (sarcoidosis), we observe granulomas without caseous necrosis, but with few inflammatory infiltrates on the periphery.

PROPOSED DIAGNOSTIC CRITERIA

Su et al5 have proposed diagnostic criteria for chilblain lupus. Their two major criteria are skin lesions in acral locations induced by exposure to cold or a drop in temperature, and evidence of lupus erythematosus in the skin lesions by histopathologic examination or immunofluorescence study. Both of these criteria must be met, plus one of three minor criteria: the coexistence of systemic lupus erythematosus or of skin lesions of discoid lupus erythematosus; response to lupus therapy; and negative results of testing for cryoglobulin and cold agglutinins.

CHILBLAIN LUPUS VS SYSTEMIC LUPUS

Chilblain lupus is an uncommon manifestation of systemic lupus erythematosus, and it is reported to occur in about 20% of patients with that condition.6 Often, the onset of chilblain lupus precedes the systemic disease. Patients with systemic lupus erythematosus and chilblain lupus do not usually present with renal disease, mucosal lesions, or central nervous system involvement. However, Raynaud phenomenon and photosensitivity have been reported to be more frequently associated with chilblain lupus.7

A disorder of peripheral circulation could be involved in the pathogenesis of chilblain lupus, and the association with Raynaud phenomenon, livedo reticularis, antiphospholipid syndrome, and changes in nailfold capillaries supports this hypothesis. Antinuclear antibody and anti-Ro/SS-A antibody are commonly detected in the serum of patients with chilblain lupus, and anti-Ro/SS-A antibody seems to be a major serologic marker of chilblain lupus in patients with systemic lupus erythematosus.7

TREATMENT

Protection from cold by physical measures is very important, as well as the use of topical or oral antibiotics if the lesions are infected. In severe cases unresponsive to topical corticosteroids, a calcium channel blocker is a good therapeutic option; antimalarials, commonly used in the treatment of lupus erythematosus, can also have a positive effect in patients with chilblain lupus.

CASE CONCLUDED

Our patient was advised to protect herself from the cold. Topical corticosteroids and oral hydroxychloroquine (200 mg/day) were prescribed, and they produced a good response. In severe cases, oral corticosteroids, etretinate (Tegison), mycophenolate (CellCept), or thalidomide (Thalomid) may be used.8

A 38-year-old woman presented with a pruriginous and erythematous lesion on her nose that appeared during periods of cold weather. She said she is completely asymptomatic during the summer months.

RTEmagicC_Arias-Santiago_ChilblainLupus_F1.gif.gif
Figure 1. The acrocyanotic lesions were covered with scales.
A physical examination revealed acrocyanotic lesions on the nose that were covered with scales (Figure 1). Laboratory testing showed increased cholesterol levels, a positive antinuclear antibody titer (1:160 or higher is positive), and a positive anti-Ro/SS-A antibody titer (1:80 or higher is positive). Tests for cryoglobulin, cold agglutinins, anti-double-stranded DNA antibody, anti-extractable nuclear antigens, C3 and C4 complement proteins, and anticardiolipin antibody were normal or negative.

RTEmagicC_52baa8f_729fig2.jpeg
Figure 2. On the left, superficial, interstitial, and deep perivascular and perifollicular dense infiltrate of lymphocytes is seen (arrows) (hematoxylin-eosin, × 4). On the right, hydropic degeneration of the basal cell layer is seen (arrow) (hematoxylin-eosin, × 40).
Histologic examination revealed degeneration of the basal layer of the dermis, with periadnexal and perivascular inflammatory infiltrates (Figure 2). On immunofluorescence testing, linear deposits of immunoglobulin M were noted at the dermoepidermal junction.

Q: What is the most likely diagnosis?

  • Lupus pernio
  • Rosacea
  • Seborrheic dermatitis
  • Chilblain lupus erythematosus
  • Lupus vulgaris

A: The diagnosis is chilblain lupus erythematosus.

The differential diagnosis of an erythematous lesion on the nose of a middle-aged woman also includes rosacea, lupus pernio, lupus vulgaris, and seborrheic dermatitis. Some of these lesions are exacerbated by cold. Usually, the diagnosis is based on clinical findings, but in some cases histologic features on biopsy study confirm the diagnosis.

Lesions of lupus pernio (sarcoidosis) remain unaltered with changes in temperature, and biopsy study usually shows granulomas without caseous necrosis with little inflammatory infiltrate at the periphery.

Rosacea usually gets worse with heat and with alcohol consumption, although it can be exacerbated by cold. Biopsy study shows a nonspecific perivascular and perifollicular lymphohistiocytic infiltrate accompanied occasionally by multinucleated cells.

Seborrheic dermatitis is a papulosquamous disorder characterized by greasy scaling over inflamed skin on the scalp, face, and trunk. Disease activity is increased in winter and spring, with remissions commonly occurring in summer. The histologic features of seborrheic dermatitis are nonspecific; in this case, the histologic features were compatible with chilblain lupus without changes of seborrheic dermatitis.

Lupus vulgaris is a chronic form of cutaneous tuberculosis characterized by redbrown papules with central atrophy. The nose and ears are usually affected. Histologically, granulomatous tubercles with epithelioid cells and caseation necrosis are usually found.

CHILBLAIN LUPUS ERYTHEMATOSUS

Pernio, or chilblain, is a localized inflammatory lesion of the skin resulting from an abnormal response to cold.1 The cutaneous lesions of chilblain may be classified as idiopathic, autoimmune-related (as in systemic lupus erythematosus, subacute cutaneous lupus), and induced by drugs such as terbinafine (Lamisil)2 or infliximab (Remicade).,3

Chilblain lupus is a rare form of cutaneous lupus erythematosus and should not be confused with lupus pernio, which is a misleading name used for a type of cutaneous sarcoidosis.4

Chilblain lupus is characterized by reddish-purple plaques in acral areas (more often the hands and feet, but also the nose and ears) that are induced by exposure to cold—unlike other lesions of lupus erythematosus, which worsen with exposure to sunlight. The main difference from the cutaneous variety of sarcoidosis (lupus pernio) is the histopathologic appearance. In patients with chilblain lupus, epidermal atrophy, perivascular and periadnexal inflammatory infiltrates, and degeneration of the basal layer are found, whereas in lupus pernio (sarcoidosis), we observe granulomas without caseous necrosis, but with few inflammatory infiltrates on the periphery.

PROPOSED DIAGNOSTIC CRITERIA

Su et al5 have proposed diagnostic criteria for chilblain lupus. Their two major criteria are skin lesions in acral locations induced by exposure to cold or a drop in temperature, and evidence of lupus erythematosus in the skin lesions by histopathologic examination or immunofluorescence study. Both of these criteria must be met, plus one of three minor criteria: the coexistence of systemic lupus erythematosus or of skin lesions of discoid lupus erythematosus; response to lupus therapy; and negative results of testing for cryoglobulin and cold agglutinins.

CHILBLAIN LUPUS VS SYSTEMIC LUPUS

Chilblain lupus is an uncommon manifestation of systemic lupus erythematosus, and it is reported to occur in about 20% of patients with that condition.6 Often, the onset of chilblain lupus precedes the systemic disease. Patients with systemic lupus erythematosus and chilblain lupus do not usually present with renal disease, mucosal lesions, or central nervous system involvement. However, Raynaud phenomenon and photosensitivity have been reported to be more frequently associated with chilblain lupus.7

A disorder of peripheral circulation could be involved in the pathogenesis of chilblain lupus, and the association with Raynaud phenomenon, livedo reticularis, antiphospholipid syndrome, and changes in nailfold capillaries supports this hypothesis. Antinuclear antibody and anti-Ro/SS-A antibody are commonly detected in the serum of patients with chilblain lupus, and anti-Ro/SS-A antibody seems to be a major serologic marker of chilblain lupus in patients with systemic lupus erythematosus.7

TREATMENT

Protection from cold by physical measures is very important, as well as the use of topical or oral antibiotics if the lesions are infected. In severe cases unresponsive to topical corticosteroids, a calcium channel blocker is a good therapeutic option; antimalarials, commonly used in the treatment of lupus erythematosus, can also have a positive effect in patients with chilblain lupus.

CASE CONCLUDED

Our patient was advised to protect herself from the cold. Topical corticosteroids and oral hydroxychloroquine (200 mg/day) were prescribed, and they produced a good response. In severe cases, oral corticosteroids, etretinate (Tegison), mycophenolate (CellCept), or thalidomide (Thalomid) may be used.8

References
  1. Simon TD, Soep JB, Hollister JR. Pernio in pediatrics. Pediatrics 2005; 116:e472e475.
  2. Bonsmann G, Schiller M, Luger TA, Ständer S. Terbinafine-induced subacute cutaneous lupus erythematosus. J Am Acad Dermatol 2001; 44:925931.
  3. Richez C, Dumoulin C, Schaeverbeke T. Infliximab induced chilblain lupus in a patient with rheumatoid arthritis. J Rheumatol 2005; 32:760761.
  4. Arias-Santiago SA, Girón-Prieto MS, Callejas-Rubio JL, Fernández-Pugnaire MA, Ortego-Centeno N. Lupus pernio or chilblain lupus?: two different entities. Chest 2009; 136:946947.
  5. Su WP, Perniciaro C, Rogers RS, White JW. Chilblain lupus erythematosus (lupus pernio): clinical review of the Mayo Clinic experience and proposal of diagnostic criteria. Cutis 1994; 54:395399.
  6. Yell JA, Mbuagbaw J, Burge SM. Cutaneous manifestations of systemic lupus erythematosus. Br J Dermatol 1996; 135:355362.
  7. Franceschini F, Calzavara-Pinton P, Quinzanini M, et al. Chilblain lupus erythematosus is associated with antibodies to SSA/Ro. Lupus 1999; 8:215219.
  8. Bouaziz JD, Barete S, Le Pelletier F, Amoura Z, Piette JC, Francès C. Cutaneous lesions of the digits in systemic lupus erythematosus: 50 cases. Lupus 2007; 16:163167.
References
  1. Simon TD, Soep JB, Hollister JR. Pernio in pediatrics. Pediatrics 2005; 116:e472e475.
  2. Bonsmann G, Schiller M, Luger TA, Ständer S. Terbinafine-induced subacute cutaneous lupus erythematosus. J Am Acad Dermatol 2001; 44:925931.
  3. Richez C, Dumoulin C, Schaeverbeke T. Infliximab induced chilblain lupus in a patient with rheumatoid arthritis. J Rheumatol 2005; 32:760761.
  4. Arias-Santiago SA, Girón-Prieto MS, Callejas-Rubio JL, Fernández-Pugnaire MA, Ortego-Centeno N. Lupus pernio or chilblain lupus?: two different entities. Chest 2009; 136:946947.
  5. Su WP, Perniciaro C, Rogers RS, White JW. Chilblain lupus erythematosus (lupus pernio): clinical review of the Mayo Clinic experience and proposal of diagnostic criteria. Cutis 1994; 54:395399.
  6. Yell JA, Mbuagbaw J, Burge SM. Cutaneous manifestations of systemic lupus erythematosus. Br J Dermatol 1996; 135:355362.
  7. Franceschini F, Calzavara-Pinton P, Quinzanini M, et al. Chilblain lupus erythematosus is associated with antibodies to SSA/Ro. Lupus 1999; 8:215219.
  8. Bouaziz JD, Barete S, Le Pelletier F, Amoura Z, Piette JC, Francès C. Cutaneous lesions of the digits in systemic lupus erythematosus: 50 cases. Lupus 2007; 16:163167.
Issue
Cleveland Clinic Journal of Medicine - 78(11)
Issue
Cleveland Clinic Journal of Medicine - 78(11)
Page Number
728-732
Page Number
728-732
Publications
Publications
Topics
Article Type
Display Headline
An erythematous plaque on the nose
Display Headline
An erythematous plaque on the nose
Sections
Disallow All Ads
Alternative CME
Article PDF Media
Image
Disable zoom
Off

Palmoplantar eruption

Article Type
Changed
Thu, 01/18/2018 - 08:37
Display Headline
Palmoplantar eruption

A 38-year-old woman presents with recurrent asymptomatic lesions on the palms and soles and on the sides of both feet. The lesions have been developing for 2 months, unaccompanied by fever or other systemic symptoms.

RTEmagicC_Arias_Santiago_PalmoplantarEruption_F1.gif.gif
Figure 1.
She has a history of episodes of arthritis of the anterior chest wall, which she has treated with nonsteroidal anti-inflammatory drugs (NSAIDs).

RTEmagicC_Arias_Santiago_PalmoplantarEruption_F2.gif.gif
Figure 2.
Physical examination reveals pustules on the palms and soles (Figure 1 and Figure 2). No lesions are noted on the oral and genital mucosae.

Laboratory tests of C-reactive protein, erythrocyte sedimentation rate, viral serologies, and antinuclear antibodies are normal. A pustule culture is negative, and a cutaneous biopsy shows parakeratosis and elongation of rete ridges, neutrophils migrating from papillary capillaries to the epidermis, and spongiform Kogoj pustule.

RTEmagicC_Arias_Santiago_PalmoplantarEruption_F3.gif.gif
Figure 3. Radionuclide uptake in the sternoclavicular joint (red arrows) and manubriosternal joint (blue arrow) was noted on bone scans.
Thoracic radiography shows increased density in the sternoclavicular joints, and a three-phase technetium-99m-labeled bone scan and gallium scan reveal radionuclide uptake in the sternoclavicular and manubriosternal joints (Figure 3). Computed tomography of the thorax and abdomen reveal no abnormalities.

Q: Which is the most likely diagnosis?

  • Pustular psoriasis
  • Impetigo contagiosa
  • Syndrome of synovitis, acne, pustulosis, hyperostosis, osteitis (SAPHO)
  • Dyshidrotic eczema
  • Acute exanthematous pustulosis (drug eruption)

A: SAPHO syndrome is the most likely diagnosis. The presence of pustules and aseptic osteitis of the anterior chest wall is compatible with SAPHO syndrome. Treatment with topical clobetasol propionate (Temovate) for pustules and NSAIDs for osteitis brought a good response.

Pustular psoriasis is an uncommon type of psoriasis characterized by erythema and pustules involving the flexural and anogenital areas. Cutaneous lesions of psoriasis vulgaris may be present before an acute pustular episode. Withdrawal of systemic corticosteroids in a patient with psoriasis has been reported as a precipitating factor.

Impetigo contagiosa is a superficial cutaneous infection characterized by an erythematous macule that evolves into a vesicle or pustule. These lesions are more common in children. Culture of the fluid usually reveals Staphylococcus aureus or S pyogenes.

Dyshidrotic eczema is a pruritic vesicular eruption of unknown cause on the palm and soles (bilateral and symmetric). The typical histologic findings are spongiotic and intraepidermal vesicles.

Acute exanthematous pustulosis is a drug-induced reaction characterized by confluent erythema, blisters, and pustules, mucous membrane erosions with fever, and lymphadenopathy. Cultures of the pustules are negative, and biopsy can help confirm the diagnosis of drug eruption.

SAPHO SYNDROME

SAPHO syndrome is a rare condition of unknown pathogenesis originally described by Chamot et al1 in 1987. The onset is usually in young adulthood, and is similar in men and women. It is characterized by synovitis, acne, pustulosis, hyperostosis, and osteitis. Of paramount importance is the finding of a non-infectious inflammatory osteitis in a patient with skin lesions.

Clinical findings: Pustules plus rheumatic pain

SAPHO syndrome must be suspected when a patient is affected by a pustular skin disease associated with rheumatic pain. If examination shows that the pain is caused by a sterile inflammation of bone or joints, the diagnosis tends to be confirmed.2

Osteoarticular involvement tends to be limited to the anterior chest wall. It may include aseptic osteitis, hyperostosis, and symmetrical arthritis. Peripheral and axial osteitis is one of the main characteristics of the syndrome and is found in around 90% of cases.

Cutaneous manifestations are present in two-thirds of patients and consist chiefly of severe acne (acne fulminans, acne conglobata, and hidradenitis suppurativa), pustular psoriasis, and palmoplantar pustulosis. Neutrophilic dermatoses associated with this syndrome include Sweet syndrome and pyoderma gangrenosum. Acne lesions are usually seen in men, whereas palmoplantar pustulosis is seen in women,3 often accompanying osteoarticular manifestations.

Radiologic findings in the spine are spondylodiskitis, osteosclerosis, sacroiliac joint involvement, and paravertebral ossification. In anterior chest wall hyperostosis, common findings are bone hypertrophy and sclerosis with a soft-tissue component. Laboratory test results are uncharacteristic, with variable signs of inflammation, and the C-reactive protein and sedimentation rate are usually elevated in the absence of leukocytosis.

Pathogenesis remains elusive

The pathogenesis of this syndrome remains elusive. Since SAPHO syndrome usually involves the axial skeleton, some investigators have suggested a possible link between the SAPHO syndrome and the seronegative spondyloarthopathies.3 It has also been related to an infection by Propionibacterium acnes and Corynebacterium species,4 which have been isolated in cultures of bone and skin lesions. However, the fact that these bacteria are contaminant agents makes their involvement in the pathogenesis of this syndrome unlikely. More recently, high concentrations of tumor necrosis factor alpha in bone specimens of patients with SAPHO syndrome have been reported, thus highlighting the central role of this cytokine in maintaining inflammation.

Treatment is to relieve symptoms

Since understanding of the pathogenesis of SAPHO syndrome is limited, a wide range of therapies has been used,5 mostly to relieve symptoms. These include NSAIDs; steroids; antibiotics; bisphosphonates such as pamidronate (Aredia) and zoledronic acid (Reclast); and immunosuppressors and immunomodulators such as methotrexate (Trexall), leflunomide (Arava), sulfasalazine (Azulfidine), cyclosporine (Sandimmune). The results with these therapies have been quite varied. Good response has been reported with tumor necrosis factor alpha blockers—infliximab (Remicade), etanercept (Enbrel), and adalimumab (Humira).6

References
  1. Chamot AM, Benhamou CL, Kahn MF, Beraneck L, Kaplan G, Prost A. Acne-pustulosis-hyperostosis-osteitis syndrome. Results of a national survey. 85 cases [in French]. Rev Rhum Mal Osteoartic 1987; 54:187196.
  2. Benhamou CL, Chamot AM, Kahn MF. Synovitis-acnepustulosis hyperostosis-osteomyelitis syndrome (SAPHO). A new syndrome among the spondyloarthropathies? Clin Exp Rheumatol 1988; 6:109112.
  3. Hayem G, Bouchaud-Chabot A, Benali K, et al. SAPHO syndrome: a long-term follow-up study of 120 cases. Semin Arthritis Rheum 1999; 29:159171.
  4. Moll C, Hernández MV, Cañete JD, et al. Ilium osteitis as the main manifestation of the SAPHO syndrome: response to infliximab therapy and review of the literature. Semin Arthritis Rheum 2008; 37:299306.
  5. Olivieri I, Padula A, Palazzi C. Pharmacological management of SAPHO syndrome. Expert Opin Investig Drugs 2006; 15:12291233.
  6. Arias-Santiago S, Sanchez-Cano D, Callejas-Rubio JL, Fernández-Pugnaire MA, Ortego-Centeno N. Adalimumab treatment for SAPHO syndrome. Acta Derm Venereol 2010; 90:301302.
Article PDF
Author and Disclosure Information

Salvador Arias-Santiago, MD
Department of Dermatology, San Cecilio University Hospital, Granada, Spain

Husein Husein El-Ahmed, MD
Department of Dermatology, San Cecilio University Hospital, Granada, Spain

José Aneiros-Fernández, MD
Department of Pathology, San Cecilio University Hospital, Granada, Spain

María Sierra Girón-Prieto, MD
Department of Dermatology, San Cecilio University Hospital, Granada, Spain

Ramón Naranjo-Sintes, PhD
Department of Dermatology, San Cecilio University Hospital, Granada, Spain

Address: Salvador Arias-Santiago, MD, San Cecilio University Hospital, Av Dr. Olóriz 16, Granada 18012, Spain; e-mail salvadorarias@hotmail.es

Issue
Cleveland Clinic Journal of Medicine - 77(10)
Publications
Topics
Page Number
729-731
Sections
Author and Disclosure Information

Salvador Arias-Santiago, MD
Department of Dermatology, San Cecilio University Hospital, Granada, Spain

Husein Husein El-Ahmed, MD
Department of Dermatology, San Cecilio University Hospital, Granada, Spain

José Aneiros-Fernández, MD
Department of Pathology, San Cecilio University Hospital, Granada, Spain

María Sierra Girón-Prieto, MD
Department of Dermatology, San Cecilio University Hospital, Granada, Spain

Ramón Naranjo-Sintes, PhD
Department of Dermatology, San Cecilio University Hospital, Granada, Spain

Address: Salvador Arias-Santiago, MD, San Cecilio University Hospital, Av Dr. Olóriz 16, Granada 18012, Spain; e-mail salvadorarias@hotmail.es

Author and Disclosure Information

Salvador Arias-Santiago, MD
Department of Dermatology, San Cecilio University Hospital, Granada, Spain

Husein Husein El-Ahmed, MD
Department of Dermatology, San Cecilio University Hospital, Granada, Spain

José Aneiros-Fernández, MD
Department of Pathology, San Cecilio University Hospital, Granada, Spain

María Sierra Girón-Prieto, MD
Department of Dermatology, San Cecilio University Hospital, Granada, Spain

Ramón Naranjo-Sintes, PhD
Department of Dermatology, San Cecilio University Hospital, Granada, Spain

Address: Salvador Arias-Santiago, MD, San Cecilio University Hospital, Av Dr. Olóriz 16, Granada 18012, Spain; e-mail salvadorarias@hotmail.es

Article PDF
Article PDF

A 38-year-old woman presents with recurrent asymptomatic lesions on the palms and soles and on the sides of both feet. The lesions have been developing for 2 months, unaccompanied by fever or other systemic symptoms.

RTEmagicC_Arias_Santiago_PalmoplantarEruption_F1.gif.gif
Figure 1.
She has a history of episodes of arthritis of the anterior chest wall, which she has treated with nonsteroidal anti-inflammatory drugs (NSAIDs).

RTEmagicC_Arias_Santiago_PalmoplantarEruption_F2.gif.gif
Figure 2.
Physical examination reveals pustules on the palms and soles (Figure 1 and Figure 2). No lesions are noted on the oral and genital mucosae.

Laboratory tests of C-reactive protein, erythrocyte sedimentation rate, viral serologies, and antinuclear antibodies are normal. A pustule culture is negative, and a cutaneous biopsy shows parakeratosis and elongation of rete ridges, neutrophils migrating from papillary capillaries to the epidermis, and spongiform Kogoj pustule.

RTEmagicC_Arias_Santiago_PalmoplantarEruption_F3.gif.gif
Figure 3. Radionuclide uptake in the sternoclavicular joint (red arrows) and manubriosternal joint (blue arrow) was noted on bone scans.
Thoracic radiography shows increased density in the sternoclavicular joints, and a three-phase technetium-99m-labeled bone scan and gallium scan reveal radionuclide uptake in the sternoclavicular and manubriosternal joints (Figure 3). Computed tomography of the thorax and abdomen reveal no abnormalities.

Q: Which is the most likely diagnosis?

  • Pustular psoriasis
  • Impetigo contagiosa
  • Syndrome of synovitis, acne, pustulosis, hyperostosis, osteitis (SAPHO)
  • Dyshidrotic eczema
  • Acute exanthematous pustulosis (drug eruption)

A: SAPHO syndrome is the most likely diagnosis. The presence of pustules and aseptic osteitis of the anterior chest wall is compatible with SAPHO syndrome. Treatment with topical clobetasol propionate (Temovate) for pustules and NSAIDs for osteitis brought a good response.

Pustular psoriasis is an uncommon type of psoriasis characterized by erythema and pustules involving the flexural and anogenital areas. Cutaneous lesions of psoriasis vulgaris may be present before an acute pustular episode. Withdrawal of systemic corticosteroids in a patient with psoriasis has been reported as a precipitating factor.

Impetigo contagiosa is a superficial cutaneous infection characterized by an erythematous macule that evolves into a vesicle or pustule. These lesions are more common in children. Culture of the fluid usually reveals Staphylococcus aureus or S pyogenes.

Dyshidrotic eczema is a pruritic vesicular eruption of unknown cause on the palm and soles (bilateral and symmetric). The typical histologic findings are spongiotic and intraepidermal vesicles.

Acute exanthematous pustulosis is a drug-induced reaction characterized by confluent erythema, blisters, and pustules, mucous membrane erosions with fever, and lymphadenopathy. Cultures of the pustules are negative, and biopsy can help confirm the diagnosis of drug eruption.

SAPHO SYNDROME

SAPHO syndrome is a rare condition of unknown pathogenesis originally described by Chamot et al1 in 1987. The onset is usually in young adulthood, and is similar in men and women. It is characterized by synovitis, acne, pustulosis, hyperostosis, and osteitis. Of paramount importance is the finding of a non-infectious inflammatory osteitis in a patient with skin lesions.

Clinical findings: Pustules plus rheumatic pain

SAPHO syndrome must be suspected when a patient is affected by a pustular skin disease associated with rheumatic pain. If examination shows that the pain is caused by a sterile inflammation of bone or joints, the diagnosis tends to be confirmed.2

Osteoarticular involvement tends to be limited to the anterior chest wall. It may include aseptic osteitis, hyperostosis, and symmetrical arthritis. Peripheral and axial osteitis is one of the main characteristics of the syndrome and is found in around 90% of cases.

Cutaneous manifestations are present in two-thirds of patients and consist chiefly of severe acne (acne fulminans, acne conglobata, and hidradenitis suppurativa), pustular psoriasis, and palmoplantar pustulosis. Neutrophilic dermatoses associated with this syndrome include Sweet syndrome and pyoderma gangrenosum. Acne lesions are usually seen in men, whereas palmoplantar pustulosis is seen in women,3 often accompanying osteoarticular manifestations.

Radiologic findings in the spine are spondylodiskitis, osteosclerosis, sacroiliac joint involvement, and paravertebral ossification. In anterior chest wall hyperostosis, common findings are bone hypertrophy and sclerosis with a soft-tissue component. Laboratory test results are uncharacteristic, with variable signs of inflammation, and the C-reactive protein and sedimentation rate are usually elevated in the absence of leukocytosis.

Pathogenesis remains elusive

The pathogenesis of this syndrome remains elusive. Since SAPHO syndrome usually involves the axial skeleton, some investigators have suggested a possible link between the SAPHO syndrome and the seronegative spondyloarthopathies.3 It has also been related to an infection by Propionibacterium acnes and Corynebacterium species,4 which have been isolated in cultures of bone and skin lesions. However, the fact that these bacteria are contaminant agents makes their involvement in the pathogenesis of this syndrome unlikely. More recently, high concentrations of tumor necrosis factor alpha in bone specimens of patients with SAPHO syndrome have been reported, thus highlighting the central role of this cytokine in maintaining inflammation.

Treatment is to relieve symptoms

Since understanding of the pathogenesis of SAPHO syndrome is limited, a wide range of therapies has been used,5 mostly to relieve symptoms. These include NSAIDs; steroids; antibiotics; bisphosphonates such as pamidronate (Aredia) and zoledronic acid (Reclast); and immunosuppressors and immunomodulators such as methotrexate (Trexall), leflunomide (Arava), sulfasalazine (Azulfidine), cyclosporine (Sandimmune). The results with these therapies have been quite varied. Good response has been reported with tumor necrosis factor alpha blockers—infliximab (Remicade), etanercept (Enbrel), and adalimumab (Humira).6

A 38-year-old woman presents with recurrent asymptomatic lesions on the palms and soles and on the sides of both feet. The lesions have been developing for 2 months, unaccompanied by fever or other systemic symptoms.

RTEmagicC_Arias_Santiago_PalmoplantarEruption_F1.gif.gif
Figure 1.
She has a history of episodes of arthritis of the anterior chest wall, which she has treated with nonsteroidal anti-inflammatory drugs (NSAIDs).

RTEmagicC_Arias_Santiago_PalmoplantarEruption_F2.gif.gif
Figure 2.
Physical examination reveals pustules on the palms and soles (Figure 1 and Figure 2). No lesions are noted on the oral and genital mucosae.

Laboratory tests of C-reactive protein, erythrocyte sedimentation rate, viral serologies, and antinuclear antibodies are normal. A pustule culture is negative, and a cutaneous biopsy shows parakeratosis and elongation of rete ridges, neutrophils migrating from papillary capillaries to the epidermis, and spongiform Kogoj pustule.

RTEmagicC_Arias_Santiago_PalmoplantarEruption_F3.gif.gif
Figure 3. Radionuclide uptake in the sternoclavicular joint (red arrows) and manubriosternal joint (blue arrow) was noted on bone scans.
Thoracic radiography shows increased density in the sternoclavicular joints, and a three-phase technetium-99m-labeled bone scan and gallium scan reveal radionuclide uptake in the sternoclavicular and manubriosternal joints (Figure 3). Computed tomography of the thorax and abdomen reveal no abnormalities.

Q: Which is the most likely diagnosis?

  • Pustular psoriasis
  • Impetigo contagiosa
  • Syndrome of synovitis, acne, pustulosis, hyperostosis, osteitis (SAPHO)
  • Dyshidrotic eczema
  • Acute exanthematous pustulosis (drug eruption)

A: SAPHO syndrome is the most likely diagnosis. The presence of pustules and aseptic osteitis of the anterior chest wall is compatible with SAPHO syndrome. Treatment with topical clobetasol propionate (Temovate) for pustules and NSAIDs for osteitis brought a good response.

Pustular psoriasis is an uncommon type of psoriasis characterized by erythema and pustules involving the flexural and anogenital areas. Cutaneous lesions of psoriasis vulgaris may be present before an acute pustular episode. Withdrawal of systemic corticosteroids in a patient with psoriasis has been reported as a precipitating factor.

Impetigo contagiosa is a superficial cutaneous infection characterized by an erythematous macule that evolves into a vesicle or pustule. These lesions are more common in children. Culture of the fluid usually reveals Staphylococcus aureus or S pyogenes.

Dyshidrotic eczema is a pruritic vesicular eruption of unknown cause on the palm and soles (bilateral and symmetric). The typical histologic findings are spongiotic and intraepidermal vesicles.

Acute exanthematous pustulosis is a drug-induced reaction characterized by confluent erythema, blisters, and pustules, mucous membrane erosions with fever, and lymphadenopathy. Cultures of the pustules are negative, and biopsy can help confirm the diagnosis of drug eruption.

SAPHO SYNDROME

SAPHO syndrome is a rare condition of unknown pathogenesis originally described by Chamot et al1 in 1987. The onset is usually in young adulthood, and is similar in men and women. It is characterized by synovitis, acne, pustulosis, hyperostosis, and osteitis. Of paramount importance is the finding of a non-infectious inflammatory osteitis in a patient with skin lesions.

Clinical findings: Pustules plus rheumatic pain

SAPHO syndrome must be suspected when a patient is affected by a pustular skin disease associated with rheumatic pain. If examination shows that the pain is caused by a sterile inflammation of bone or joints, the diagnosis tends to be confirmed.2

Osteoarticular involvement tends to be limited to the anterior chest wall. It may include aseptic osteitis, hyperostosis, and symmetrical arthritis. Peripheral and axial osteitis is one of the main characteristics of the syndrome and is found in around 90% of cases.

Cutaneous manifestations are present in two-thirds of patients and consist chiefly of severe acne (acne fulminans, acne conglobata, and hidradenitis suppurativa), pustular psoriasis, and palmoplantar pustulosis. Neutrophilic dermatoses associated with this syndrome include Sweet syndrome and pyoderma gangrenosum. Acne lesions are usually seen in men, whereas palmoplantar pustulosis is seen in women,3 often accompanying osteoarticular manifestations.

Radiologic findings in the spine are spondylodiskitis, osteosclerosis, sacroiliac joint involvement, and paravertebral ossification. In anterior chest wall hyperostosis, common findings are bone hypertrophy and sclerosis with a soft-tissue component. Laboratory test results are uncharacteristic, with variable signs of inflammation, and the C-reactive protein and sedimentation rate are usually elevated in the absence of leukocytosis.

Pathogenesis remains elusive

The pathogenesis of this syndrome remains elusive. Since SAPHO syndrome usually involves the axial skeleton, some investigators have suggested a possible link between the SAPHO syndrome and the seronegative spondyloarthopathies.3 It has also been related to an infection by Propionibacterium acnes and Corynebacterium species,4 which have been isolated in cultures of bone and skin lesions. However, the fact that these bacteria are contaminant agents makes their involvement in the pathogenesis of this syndrome unlikely. More recently, high concentrations of tumor necrosis factor alpha in bone specimens of patients with SAPHO syndrome have been reported, thus highlighting the central role of this cytokine in maintaining inflammation.

Treatment is to relieve symptoms

Since understanding of the pathogenesis of SAPHO syndrome is limited, a wide range of therapies has been used,5 mostly to relieve symptoms. These include NSAIDs; steroids; antibiotics; bisphosphonates such as pamidronate (Aredia) and zoledronic acid (Reclast); and immunosuppressors and immunomodulators such as methotrexate (Trexall), leflunomide (Arava), sulfasalazine (Azulfidine), cyclosporine (Sandimmune). The results with these therapies have been quite varied. Good response has been reported with tumor necrosis factor alpha blockers—infliximab (Remicade), etanercept (Enbrel), and adalimumab (Humira).6

References
  1. Chamot AM, Benhamou CL, Kahn MF, Beraneck L, Kaplan G, Prost A. Acne-pustulosis-hyperostosis-osteitis syndrome. Results of a national survey. 85 cases [in French]. Rev Rhum Mal Osteoartic 1987; 54:187196.
  2. Benhamou CL, Chamot AM, Kahn MF. Synovitis-acnepustulosis hyperostosis-osteomyelitis syndrome (SAPHO). A new syndrome among the spondyloarthropathies? Clin Exp Rheumatol 1988; 6:109112.
  3. Hayem G, Bouchaud-Chabot A, Benali K, et al. SAPHO syndrome: a long-term follow-up study of 120 cases. Semin Arthritis Rheum 1999; 29:159171.
  4. Moll C, Hernández MV, Cañete JD, et al. Ilium osteitis as the main manifestation of the SAPHO syndrome: response to infliximab therapy and review of the literature. Semin Arthritis Rheum 2008; 37:299306.
  5. Olivieri I, Padula A, Palazzi C. Pharmacological management of SAPHO syndrome. Expert Opin Investig Drugs 2006; 15:12291233.
  6. Arias-Santiago S, Sanchez-Cano D, Callejas-Rubio JL, Fernández-Pugnaire MA, Ortego-Centeno N. Adalimumab treatment for SAPHO syndrome. Acta Derm Venereol 2010; 90:301302.
References
  1. Chamot AM, Benhamou CL, Kahn MF, Beraneck L, Kaplan G, Prost A. Acne-pustulosis-hyperostosis-osteitis syndrome. Results of a national survey. 85 cases [in French]. Rev Rhum Mal Osteoartic 1987; 54:187196.
  2. Benhamou CL, Chamot AM, Kahn MF. Synovitis-acnepustulosis hyperostosis-osteomyelitis syndrome (SAPHO). A new syndrome among the spondyloarthropathies? Clin Exp Rheumatol 1988; 6:109112.
  3. Hayem G, Bouchaud-Chabot A, Benali K, et al. SAPHO syndrome: a long-term follow-up study of 120 cases. Semin Arthritis Rheum 1999; 29:159171.
  4. Moll C, Hernández MV, Cañete JD, et al. Ilium osteitis as the main manifestation of the SAPHO syndrome: response to infliximab therapy and review of the literature. Semin Arthritis Rheum 2008; 37:299306.
  5. Olivieri I, Padula A, Palazzi C. Pharmacological management of SAPHO syndrome. Expert Opin Investig Drugs 2006; 15:12291233.
  6. Arias-Santiago S, Sanchez-Cano D, Callejas-Rubio JL, Fernández-Pugnaire MA, Ortego-Centeno N. Adalimumab treatment for SAPHO syndrome. Acta Derm Venereol 2010; 90:301302.
Issue
Cleveland Clinic Journal of Medicine - 77(10)
Issue
Cleveland Clinic Journal of Medicine - 77(10)
Page Number
729-731
Page Number
729-731
Publications
Publications
Topics
Article Type
Display Headline
Palmoplantar eruption
Display Headline
Palmoplantar eruption
Sections
Disallow All Ads
Alternative CME
Article PDF Media
Image
Disable zoom
Off

An ulcerated plaque on the hand

Article Type
Changed
Wed, 01/17/2018 - 11:49
Display Headline
An ulcerated plaque on the hand

RTEmagicC_Husein-ElAhmed_UlceratedPlaque_F1.gif.gif
Figure 1.
A 73-year-old farmer has a lesion on the dorsum of his right hand that bleeds intermittently. It began 2 years ago as a small itchy papule. It later ulcerated and has gradually increased in size over the past 2 years.

RTEmagicC_Husein-ElAhmed_UlceratedPlaque_F2.gif.gif
Figure 2.
On examination, the lesion is a nontender, ulcerated plaque, 4 cm by 5 cm, with hard and raised edges (Figure 1 and Figure 2) and with evidence of chronic photodamage. No local adenopathy is noted.

Q: Which is the most likely diagnosis?

  • Blastomycosis
  • Keratoacanthoma
  • Basal cell carcinoma
  • Squamous cell carcinoma
  • Extramammary Paget disease

A: Squamous cell carcinoma is the most likely diagnosis. The lesions may take the form of a patch, plaque, or nodule, sometimes with scaling or an ulcerated center. The borders often are irregular, fleshy, and bleed easily.

Blastomycosis is a rare fungal infection caused by inhaling the spores of the fungus Blastomyces dermatitidis. Skin features begin as papules, pustules, or subcutaneous nodules. Over a period of months, lesions heal to form raised wart-like scars, which are often irreversible.

Keratoacanthoma is a benign nonmelanomatous cancer that grows rapidly and remits spontaneously. This diagnosis is unlikely in our patient, whose lesion evolved over 2 years.

Basal cell carcinoma has a waxy, translucent, or pearly appearance, without a keratotic component. Telangiectasias are common. Unlike squamous cell carcinomas, the edges of basal cell lesions are clear rather than fleshy.

Extramammary Paget disease usually manifests as intense pruritus in affected areas, which appear as chronic intertrigo or nonresolving eczema. The dorsum of the hand is a very rare location for this disease. Given our patient’s presentation, extrammary Paget disease is an unlikely diagnosis.

CHRONIC SUN EXPOSURE IS A KEY RISK FACTOR

Our patient has worked outdoors for many years, and chronic sun exposure is a key risk factor for squamous cell carcinoma.1

Squamous cell carcinoma is a nonmelanomatous skin cancer arising from the more superficial layers of keratinocytes. Unlike basal cell carcinoma, it can metastasize,2 so the diagnosis must be made as soon as possible.

The key to diagnosis is a high degree of suspicion. Patients usually present with chronic sun-damaged skin with lesions as actinic keratoses or keratin horns. Squamous cell carcinoma appears as a thick, adherent scale that does not heal and may intermittently bleed. Because it spreads into the dermis, it can appear like an ulcer, with hard, raised edges, as in our patient.

TREATMENT AND PREVENTION

The standard effective treatment is complete surgical excision.1 Nonsurgical treatments include curettage plus cautery, cryosurgery, radiotherapy, photodynamic therapy, and imiquimod (Aldara) 5% cream. Minimizing sun exposure and regular checkups are important preventive measures.

Our patient underwent total surgical excision of the lesion. Due to the size of the lesion, a skin graft was required and was obtained from the abdomen. No relapse was observed after 1 year of follow-up.

References
  1. Alam M, Ratner D. Cutaneous squamous-cell carcinoma. N Engl J Med 2001; 344:975983.
  2. Rowe DE, Carroll RJ, Day CL. Prognostic factors for local recurrence, metastasis and survival rates in squamous cell carcinoma of the skin, ear, and lip: implications for treatment modality selection. J Am Acad Dermatol 1992; 26:976990.
Article PDF
Author and Disclosure Information

Husein Husein El-Ahmed, MD
Department of Dermatology, University Hospital, Granada, Spain

José Aneiros-Fernández, MD
Department of Pathology, University Hospital, Granada, Spain

Salvador Arias-Santiago, MD
Department of Dermatology, University Hospital, Granada, Spain

Rosa Ortego Del Olmo, MD
Department of Dermatology, University Hospital, Granada, Spain

Ramón Naranjo-Sintes, PhD
Department of Dermatology, University Hospital, Granada, Spain

Address: Husein Husein El-Ahmed, MD, Department of Dermatology, San Cecilio University Hospital, Avenida Dr. Olóriz, 18012 Granada, Spain; e-mail huseinelahmed@hotmail.com

Issue
Cleveland Clinic Journal of Medicine - 77(9)
Publications
Topics
Page Number
585-586
Sections
Author and Disclosure Information

Husein Husein El-Ahmed, MD
Department of Dermatology, University Hospital, Granada, Spain

José Aneiros-Fernández, MD
Department of Pathology, University Hospital, Granada, Spain

Salvador Arias-Santiago, MD
Department of Dermatology, University Hospital, Granada, Spain

Rosa Ortego Del Olmo, MD
Department of Dermatology, University Hospital, Granada, Spain

Ramón Naranjo-Sintes, PhD
Department of Dermatology, University Hospital, Granada, Spain

Address: Husein Husein El-Ahmed, MD, Department of Dermatology, San Cecilio University Hospital, Avenida Dr. Olóriz, 18012 Granada, Spain; e-mail huseinelahmed@hotmail.com

Author and Disclosure Information

Husein Husein El-Ahmed, MD
Department of Dermatology, University Hospital, Granada, Spain

José Aneiros-Fernández, MD
Department of Pathology, University Hospital, Granada, Spain

Salvador Arias-Santiago, MD
Department of Dermatology, University Hospital, Granada, Spain

Rosa Ortego Del Olmo, MD
Department of Dermatology, University Hospital, Granada, Spain

Ramón Naranjo-Sintes, PhD
Department of Dermatology, University Hospital, Granada, Spain

Address: Husein Husein El-Ahmed, MD, Department of Dermatology, San Cecilio University Hospital, Avenida Dr. Olóriz, 18012 Granada, Spain; e-mail huseinelahmed@hotmail.com

Article PDF
Article PDF

RTEmagicC_Husein-ElAhmed_UlceratedPlaque_F1.gif.gif
Figure 1.
A 73-year-old farmer has a lesion on the dorsum of his right hand that bleeds intermittently. It began 2 years ago as a small itchy papule. It later ulcerated and has gradually increased in size over the past 2 years.

RTEmagicC_Husein-ElAhmed_UlceratedPlaque_F2.gif.gif
Figure 2.
On examination, the lesion is a nontender, ulcerated plaque, 4 cm by 5 cm, with hard and raised edges (Figure 1 and Figure 2) and with evidence of chronic photodamage. No local adenopathy is noted.

Q: Which is the most likely diagnosis?

  • Blastomycosis
  • Keratoacanthoma
  • Basal cell carcinoma
  • Squamous cell carcinoma
  • Extramammary Paget disease

A: Squamous cell carcinoma is the most likely diagnosis. The lesions may take the form of a patch, plaque, or nodule, sometimes with scaling or an ulcerated center. The borders often are irregular, fleshy, and bleed easily.

Blastomycosis is a rare fungal infection caused by inhaling the spores of the fungus Blastomyces dermatitidis. Skin features begin as papules, pustules, or subcutaneous nodules. Over a period of months, lesions heal to form raised wart-like scars, which are often irreversible.

Keratoacanthoma is a benign nonmelanomatous cancer that grows rapidly and remits spontaneously. This diagnosis is unlikely in our patient, whose lesion evolved over 2 years.

Basal cell carcinoma has a waxy, translucent, or pearly appearance, without a keratotic component. Telangiectasias are common. Unlike squamous cell carcinomas, the edges of basal cell lesions are clear rather than fleshy.

Extramammary Paget disease usually manifests as intense pruritus in affected areas, which appear as chronic intertrigo or nonresolving eczema. The dorsum of the hand is a very rare location for this disease. Given our patient’s presentation, extrammary Paget disease is an unlikely diagnosis.

CHRONIC SUN EXPOSURE IS A KEY RISK FACTOR

Our patient has worked outdoors for many years, and chronic sun exposure is a key risk factor for squamous cell carcinoma.1

Squamous cell carcinoma is a nonmelanomatous skin cancer arising from the more superficial layers of keratinocytes. Unlike basal cell carcinoma, it can metastasize,2 so the diagnosis must be made as soon as possible.

The key to diagnosis is a high degree of suspicion. Patients usually present with chronic sun-damaged skin with lesions as actinic keratoses or keratin horns. Squamous cell carcinoma appears as a thick, adherent scale that does not heal and may intermittently bleed. Because it spreads into the dermis, it can appear like an ulcer, with hard, raised edges, as in our patient.

TREATMENT AND PREVENTION

The standard effective treatment is complete surgical excision.1 Nonsurgical treatments include curettage plus cautery, cryosurgery, radiotherapy, photodynamic therapy, and imiquimod (Aldara) 5% cream. Minimizing sun exposure and regular checkups are important preventive measures.

Our patient underwent total surgical excision of the lesion. Due to the size of the lesion, a skin graft was required and was obtained from the abdomen. No relapse was observed after 1 year of follow-up.

RTEmagicC_Husein-ElAhmed_UlceratedPlaque_F1.gif.gif
Figure 1.
A 73-year-old farmer has a lesion on the dorsum of his right hand that bleeds intermittently. It began 2 years ago as a small itchy papule. It later ulcerated and has gradually increased in size over the past 2 years.

RTEmagicC_Husein-ElAhmed_UlceratedPlaque_F2.gif.gif
Figure 2.
On examination, the lesion is a nontender, ulcerated plaque, 4 cm by 5 cm, with hard and raised edges (Figure 1 and Figure 2) and with evidence of chronic photodamage. No local adenopathy is noted.

Q: Which is the most likely diagnosis?

  • Blastomycosis
  • Keratoacanthoma
  • Basal cell carcinoma
  • Squamous cell carcinoma
  • Extramammary Paget disease

A: Squamous cell carcinoma is the most likely diagnosis. The lesions may take the form of a patch, plaque, or nodule, sometimes with scaling or an ulcerated center. The borders often are irregular, fleshy, and bleed easily.

Blastomycosis is a rare fungal infection caused by inhaling the spores of the fungus Blastomyces dermatitidis. Skin features begin as papules, pustules, or subcutaneous nodules. Over a period of months, lesions heal to form raised wart-like scars, which are often irreversible.

Keratoacanthoma is a benign nonmelanomatous cancer that grows rapidly and remits spontaneously. This diagnosis is unlikely in our patient, whose lesion evolved over 2 years.

Basal cell carcinoma has a waxy, translucent, or pearly appearance, without a keratotic component. Telangiectasias are common. Unlike squamous cell carcinomas, the edges of basal cell lesions are clear rather than fleshy.

Extramammary Paget disease usually manifests as intense pruritus in affected areas, which appear as chronic intertrigo or nonresolving eczema. The dorsum of the hand is a very rare location for this disease. Given our patient’s presentation, extrammary Paget disease is an unlikely diagnosis.

CHRONIC SUN EXPOSURE IS A KEY RISK FACTOR

Our patient has worked outdoors for many years, and chronic sun exposure is a key risk factor for squamous cell carcinoma.1

Squamous cell carcinoma is a nonmelanomatous skin cancer arising from the more superficial layers of keratinocytes. Unlike basal cell carcinoma, it can metastasize,2 so the diagnosis must be made as soon as possible.

The key to diagnosis is a high degree of suspicion. Patients usually present with chronic sun-damaged skin with lesions as actinic keratoses or keratin horns. Squamous cell carcinoma appears as a thick, adherent scale that does not heal and may intermittently bleed. Because it spreads into the dermis, it can appear like an ulcer, with hard, raised edges, as in our patient.

TREATMENT AND PREVENTION

The standard effective treatment is complete surgical excision.1 Nonsurgical treatments include curettage plus cautery, cryosurgery, radiotherapy, photodynamic therapy, and imiquimod (Aldara) 5% cream. Minimizing sun exposure and regular checkups are important preventive measures.

Our patient underwent total surgical excision of the lesion. Due to the size of the lesion, a skin graft was required and was obtained from the abdomen. No relapse was observed after 1 year of follow-up.

References
  1. Alam M, Ratner D. Cutaneous squamous-cell carcinoma. N Engl J Med 2001; 344:975983.
  2. Rowe DE, Carroll RJ, Day CL. Prognostic factors for local recurrence, metastasis and survival rates in squamous cell carcinoma of the skin, ear, and lip: implications for treatment modality selection. J Am Acad Dermatol 1992; 26:976990.
References
  1. Alam M, Ratner D. Cutaneous squamous-cell carcinoma. N Engl J Med 2001; 344:975983.
  2. Rowe DE, Carroll RJ, Day CL. Prognostic factors for local recurrence, metastasis and survival rates in squamous cell carcinoma of the skin, ear, and lip: implications for treatment modality selection. J Am Acad Dermatol 1992; 26:976990.
Issue
Cleveland Clinic Journal of Medicine - 77(9)
Issue
Cleveland Clinic Journal of Medicine - 77(9)
Page Number
585-586
Page Number
585-586
Publications
Publications
Topics
Article Type
Display Headline
An ulcerated plaque on the hand
Display Headline
An ulcerated plaque on the hand
Sections
Disallow All Ads
Alternative CME
Article PDF Media
Image
Disable zoom
Off

Painful red nodule on the right hand

Article Type
Changed
Wed, 01/17/2018 - 11:50
Display Headline
Painful red nodule on the right hand

A 46-year-old healthy man presents with a 15-day history of a tender subcutaneous nodule on the dorsum of the right hand that appeared after cleaning his aquarium. He has no fever or systemic symptoms. For 2 weeks he has been taking amoxicillin-clavulanate (Augmentin) and metronidazole (Flagyl), but without an adequate response.

RTEmagicC_Arias-Santiago_MMarinum_F1.gif.gif
Figure 1. A red nodule with central ulceration on the right hand.
RTEmagicC_Arias-Santiago_MMarinum_F2.gif.gif
Figure 2. A nodule on the patient’s forearm with a sporotrichoid distribution. Locoregional lymph nodes were not palpable.
On physical examination, the nodule is painful and has central ulceration (Figure 1). Another two nodules are noted on the forearm, with a sporotrichoid distribution (Figure 2). Locoregional lymph nodes are not palpable.

RTEmagicC_Arias-Santiago_MMarinum_F3.gif.gif
Figure 3. Histologic testing revealed a mixed infiltrate with lymphocytes, neutrophils, and histiocytes (hematoxylin-eosin, × 40).
Laboratory testing (hemography, biochemistry panel, coagulation test, and C-reactive protein) are normal. Routine bacterial cultures of blood and wound drainage are repeatedly negative. Histologic study (Figure 3) reveals signs of acute and chronic inflammation (a mixed infiltrate with lymphocytes, neutrophils, and histiocytes) with abundant bacilli by Ziehl-Neelsen and Giemsa stains.

Q: Which is the most likely diagnosis?

  • Tuberculosis verrucosa cutis
  • Sporotrichosis
  • Nontuberculous mycobacterial infection
  • Leishmaniasis
  • Nocardiosis

A: The correct answer is nontuberculous mycobacterial infection. Mycobacterium marinum was subsequently isolated from culture (Löwenstein-Jensen medium) of wound drainage.

Tuberculosis verrucosa cutis is an indolent, warty plaque that occurs after direct inoculation of M tuberculosis into the skin of someone previously infected with M tuberculosis. It can result from accidental exposure to tuberculous tissue in high-risk groups such as laboratory workers, physicians, and pathologists.

Sporothrix schenckii is the dimorphic fungus that causes sporotrichosis. Activities associated with acquisition of sporotrichosis include gardening (rose-gardener's disease), landscaping, farming, berry-picking, horticulture, and carpentry. The characteristic infection involves suppurating subcutaneous nodules that progress proximally along lymphatic channels (lymphocutaneous sporotrichosis).

Leishmaniasis is a disease caused by the protozoa of the Leishmania species, which is transmitted by the bite of a female sandfly. Initially, the lesion is a small, red papule up to 2 cm in diameter. Over several weeks, the papule becomes darker, develops a crust in the center, and eventually ulcerates to present a typical appearance of an ulcer with raised edges and surrounding dusky red skin.

Nocardia is a genus of filamentous grampositive bacteria that stains acid-fast, just as M marinum does. Nocardia asteroides primarily affects the lungs and can disseminate systemically. However, Nocardia brasiliensis is often associated with sporotrichoid-spreading subcutaneous nodules.

CLINICAL PRESENTATION AND DIAGNOSIS

M marinum is a genus of nontuberculous mycobacteria that usually causes disease in fish but can cause human skin infection by penetrating through a break in the skin. It can spread to deeper structures, resulting in tenosynovitis, arthritis, or osteomyelitis.1 The disease caused by M marinum is sometimes called “swimming pool granuloma” or “fish-tank granuloma.”2

In this case, the patient probably acquired the infection while cleaning his home aquarium,3 so asking about contact with pet fish is very important for the clinical diagnosis.

Lesions are usually localized to the site of the inoculation, with a predilection for areas predisposed to trauma, such as the hands and the fingers, after an incubation period of 2 to 8 weeks. Lesions are self-limited and usually appear as pruriginous bluish-red papules or pustules that may increase in size to form a verrucous or violaceous plaque or nodule. Superficial lesions often undergo central ulceration. Disseminated infection is rare and occurs mainly in patients who are immunocompromised because of renal transplant, systemic lupus erythematosus, chronic steroid therapy, or anti-tumor necrosis factor alpha therapy.4 In such cases, infection can be life-threatening.

On histopathologic study, a nonspecific inflammation consisting of a mixed infiltrate with lymphocytes, neutrophils, and histiocytes is usually observed. Sometimes a granulomatous inflammatory infiltrate mimicking tuberculoid granuloma may be noted. M marinum is an aerobic, nonmotile acid-fast bacillus. It grows at 30°C to 32°C on Löwenstein-Jensen medium in 2 to 5 weeks, but cultures are rarely positive. Polymerase chain reaction and enzyme-linked immunosorbent assays can help identify the organism. Other diagnoses to be considered are leishmaniasis, tuberculosis verrucosa cutis, blastomycosis, histoplasmosis, and sporotrichosis.

TREATMENT

Treatment of M marinum infection is not based on any specific criteria. Spontaneous resolution in 24 to 36 months has been described. 5 Antibiotics are the first-line therapy, and sometimes surgical treatment may be necessary for deeper infection with necrotic tissue. Cryotherapy, electrode therapy, and irradiation have been reported to be effective.

Few studies have been conducted to determine the first-line antibiotic treatment for M marinum infection. In this case, treatment with sulfamethoxazole-trimethoprim6 (Bactrim) resulted in resolution of the lesions in 6 months. Other antibiotics often used as monotherapy are tetracycline, minocycline (Minocin), and doxycycline (Vibramycin) 100 mg twice daily for 3 months.7 However, treatment failure has been described with many antibiotics.

Multidrug therapy is usually prescribed to minimize the risk of resistance. A combination of clarithromycin (Biaxin), streptomycin, and ethionamide (Trecator) has been prescribed successfully.3 Another combined regimen—clarithromycin 500 mg twice a day, rifampin (Rifadin) 600 mg daily, and ethambutol (Myambutol) 25 mg/kg daily—has been shown to be effective.5 Also, rifampin 600 mg daily and ethambutol 15 to 25 mg/kg/day were effective in a patient with a sporotrichoid M marinum infection associated with infliximab (Remicade). 8 The duration of treatment ranged between 2 and 6 months, or up to 2 months after the disappearance of the cutaneous lesions.

Clinical awareness of M marinum infection is important so that the diagnosis can be made and appropriate therapy can be initiated promptly, because some cases with disseminated disease and serious complications have been described.9

References
  1. Lam A, Toma W, Schlesinger N. Mycobacterium marinum arthritis mimicking rheumatoid arthritis. J Rheumatol 2006; 33:817819.
  2. Lewis FM, Marsh BJ, von Reyn CF. Fish tank exposure and cutaneous infections due to Mycobacterium marinum: tuberculin skin testing, treatment, and prevention. Clin Infect Dis 2003; 37:390397.
  3. Dodiuk-Gad R, Dyachenko P, Ziv M, et al. Nontuberculous mycobacterial infections of the skin: a retrospective study of 25 cases. J Am Acad Dermatol 2007; 57:413420.
  4. Streit M, Böhlen LM, Hunziker T, et al. Disseminated Mycobacterium marinum infection with extensive cutaneous eruption and bacteremia in an immunocompromised patient. Eur J Dermatol 2006; 16:7983.
  5. Jogi R, Tyring SK. Therapy of nontuberculous mycobacterial infections. Dermatol Ther 2004; 17:491498.
  6. Alinovi A, Vecchini F, Bassissi P. Sporothricoid mycobacterial infection. A case report. Acta Derm Venereol 1993; 73:146147.
  7. Mahaisavariya P, Chaiprasert A, Khemngern S, et al. Nontuberculous mycobacterial skin infections: clinical and bacteriological studies. J Med Assoc Thai 2003; 86:5260.
  8. Rallis E, Koumantaki-Mathioudaki E, Frangoulis E, Chatziolou E, Katsambas A. Severe sporotrichoid fish tank granuloma following infliximab therapy. Am J Clin Dermatol 2007; 8:385388.
  9. Imakado S, Kojima Y, Hiyoshi T, Morimoto S. Disseminated Mycobacterium marinum infection in a patient with diabetic nephropathy. Diabetes Res Clin Pract 2009; 83:e35e36.
Article PDF
Author and Disclosure Information

Salvador Arias-Santiago, MD
Department of Dermatology, San Cecilio University Hospital, Granada, Spain

José Aneiros-Fernández, MD
Department of Pathology, San Cecilio University Hospital, Granada, Spain

Husein Husein El-Ahmed, MD
Department of Dermatology, San Cecilio University Hospital, Granada, Spain

María Sierra Girón-Prieto, MD
Department of Dermatology, San Cecilio University Hospital, Granada, Spain

Leopoldo Muñoz-Medina, PhD
Department of Internal Medicine, San Cecilio University Hospital, Granada, Spain

Ramón Naranjo-Sintes, PhD
Department of Dermatology, San Cecilio University Hospital, Granada, Spain

Address: Salvador Arias-Santiago, MD, San Cecilio University Hospital, Av Dr Olóriz 16, Granada 18012, Spain; e-mail salvadorarias@hotmail.es

Issue
Cleveland Clinic Journal of Medicine - 77(8)
Publications
Topics
Page Number
512-515
Sections
Author and Disclosure Information

Salvador Arias-Santiago, MD
Department of Dermatology, San Cecilio University Hospital, Granada, Spain

José Aneiros-Fernández, MD
Department of Pathology, San Cecilio University Hospital, Granada, Spain

Husein Husein El-Ahmed, MD
Department of Dermatology, San Cecilio University Hospital, Granada, Spain

María Sierra Girón-Prieto, MD
Department of Dermatology, San Cecilio University Hospital, Granada, Spain

Leopoldo Muñoz-Medina, PhD
Department of Internal Medicine, San Cecilio University Hospital, Granada, Spain

Ramón Naranjo-Sintes, PhD
Department of Dermatology, San Cecilio University Hospital, Granada, Spain

Address: Salvador Arias-Santiago, MD, San Cecilio University Hospital, Av Dr Olóriz 16, Granada 18012, Spain; e-mail salvadorarias@hotmail.es

Author and Disclosure Information

Salvador Arias-Santiago, MD
Department of Dermatology, San Cecilio University Hospital, Granada, Spain

José Aneiros-Fernández, MD
Department of Pathology, San Cecilio University Hospital, Granada, Spain

Husein Husein El-Ahmed, MD
Department of Dermatology, San Cecilio University Hospital, Granada, Spain

María Sierra Girón-Prieto, MD
Department of Dermatology, San Cecilio University Hospital, Granada, Spain

Leopoldo Muñoz-Medina, PhD
Department of Internal Medicine, San Cecilio University Hospital, Granada, Spain

Ramón Naranjo-Sintes, PhD
Department of Dermatology, San Cecilio University Hospital, Granada, Spain

Address: Salvador Arias-Santiago, MD, San Cecilio University Hospital, Av Dr Olóriz 16, Granada 18012, Spain; e-mail salvadorarias@hotmail.es

Article PDF
Article PDF

A 46-year-old healthy man presents with a 15-day history of a tender subcutaneous nodule on the dorsum of the right hand that appeared after cleaning his aquarium. He has no fever or systemic symptoms. For 2 weeks he has been taking amoxicillin-clavulanate (Augmentin) and metronidazole (Flagyl), but without an adequate response.

RTEmagicC_Arias-Santiago_MMarinum_F1.gif.gif
Figure 1. A red nodule with central ulceration on the right hand.
RTEmagicC_Arias-Santiago_MMarinum_F2.gif.gif
Figure 2. A nodule on the patient’s forearm with a sporotrichoid distribution. Locoregional lymph nodes were not palpable.
On physical examination, the nodule is painful and has central ulceration (Figure 1). Another two nodules are noted on the forearm, with a sporotrichoid distribution (Figure 2). Locoregional lymph nodes are not palpable.

RTEmagicC_Arias-Santiago_MMarinum_F3.gif.gif
Figure 3. Histologic testing revealed a mixed infiltrate with lymphocytes, neutrophils, and histiocytes (hematoxylin-eosin, × 40).
Laboratory testing (hemography, biochemistry panel, coagulation test, and C-reactive protein) are normal. Routine bacterial cultures of blood and wound drainage are repeatedly negative. Histologic study (Figure 3) reveals signs of acute and chronic inflammation (a mixed infiltrate with lymphocytes, neutrophils, and histiocytes) with abundant bacilli by Ziehl-Neelsen and Giemsa stains.

Q: Which is the most likely diagnosis?

  • Tuberculosis verrucosa cutis
  • Sporotrichosis
  • Nontuberculous mycobacterial infection
  • Leishmaniasis
  • Nocardiosis

A: The correct answer is nontuberculous mycobacterial infection. Mycobacterium marinum was subsequently isolated from culture (Löwenstein-Jensen medium) of wound drainage.

Tuberculosis verrucosa cutis is an indolent, warty plaque that occurs after direct inoculation of M tuberculosis into the skin of someone previously infected with M tuberculosis. It can result from accidental exposure to tuberculous tissue in high-risk groups such as laboratory workers, physicians, and pathologists.

Sporothrix schenckii is the dimorphic fungus that causes sporotrichosis. Activities associated with acquisition of sporotrichosis include gardening (rose-gardener's disease), landscaping, farming, berry-picking, horticulture, and carpentry. The characteristic infection involves suppurating subcutaneous nodules that progress proximally along lymphatic channels (lymphocutaneous sporotrichosis).

Leishmaniasis is a disease caused by the protozoa of the Leishmania species, which is transmitted by the bite of a female sandfly. Initially, the lesion is a small, red papule up to 2 cm in diameter. Over several weeks, the papule becomes darker, develops a crust in the center, and eventually ulcerates to present a typical appearance of an ulcer with raised edges and surrounding dusky red skin.

Nocardia is a genus of filamentous grampositive bacteria that stains acid-fast, just as M marinum does. Nocardia asteroides primarily affects the lungs and can disseminate systemically. However, Nocardia brasiliensis is often associated with sporotrichoid-spreading subcutaneous nodules.

CLINICAL PRESENTATION AND DIAGNOSIS

M marinum is a genus of nontuberculous mycobacteria that usually causes disease in fish but can cause human skin infection by penetrating through a break in the skin. It can spread to deeper structures, resulting in tenosynovitis, arthritis, or osteomyelitis.1 The disease caused by M marinum is sometimes called “swimming pool granuloma” or “fish-tank granuloma.”2

In this case, the patient probably acquired the infection while cleaning his home aquarium,3 so asking about contact with pet fish is very important for the clinical diagnosis.

Lesions are usually localized to the site of the inoculation, with a predilection for areas predisposed to trauma, such as the hands and the fingers, after an incubation period of 2 to 8 weeks. Lesions are self-limited and usually appear as pruriginous bluish-red papules or pustules that may increase in size to form a verrucous or violaceous plaque or nodule. Superficial lesions often undergo central ulceration. Disseminated infection is rare and occurs mainly in patients who are immunocompromised because of renal transplant, systemic lupus erythematosus, chronic steroid therapy, or anti-tumor necrosis factor alpha therapy.4 In such cases, infection can be life-threatening.

On histopathologic study, a nonspecific inflammation consisting of a mixed infiltrate with lymphocytes, neutrophils, and histiocytes is usually observed. Sometimes a granulomatous inflammatory infiltrate mimicking tuberculoid granuloma may be noted. M marinum is an aerobic, nonmotile acid-fast bacillus. It grows at 30°C to 32°C on Löwenstein-Jensen medium in 2 to 5 weeks, but cultures are rarely positive. Polymerase chain reaction and enzyme-linked immunosorbent assays can help identify the organism. Other diagnoses to be considered are leishmaniasis, tuberculosis verrucosa cutis, blastomycosis, histoplasmosis, and sporotrichosis.

TREATMENT

Treatment of M marinum infection is not based on any specific criteria. Spontaneous resolution in 24 to 36 months has been described. 5 Antibiotics are the first-line therapy, and sometimes surgical treatment may be necessary for deeper infection with necrotic tissue. Cryotherapy, electrode therapy, and irradiation have been reported to be effective.

Few studies have been conducted to determine the first-line antibiotic treatment for M marinum infection. In this case, treatment with sulfamethoxazole-trimethoprim6 (Bactrim) resulted in resolution of the lesions in 6 months. Other antibiotics often used as monotherapy are tetracycline, minocycline (Minocin), and doxycycline (Vibramycin) 100 mg twice daily for 3 months.7 However, treatment failure has been described with many antibiotics.

Multidrug therapy is usually prescribed to minimize the risk of resistance. A combination of clarithromycin (Biaxin), streptomycin, and ethionamide (Trecator) has been prescribed successfully.3 Another combined regimen—clarithromycin 500 mg twice a day, rifampin (Rifadin) 600 mg daily, and ethambutol (Myambutol) 25 mg/kg daily—has been shown to be effective.5 Also, rifampin 600 mg daily and ethambutol 15 to 25 mg/kg/day were effective in a patient with a sporotrichoid M marinum infection associated with infliximab (Remicade). 8 The duration of treatment ranged between 2 and 6 months, or up to 2 months after the disappearance of the cutaneous lesions.

Clinical awareness of M marinum infection is important so that the diagnosis can be made and appropriate therapy can be initiated promptly, because some cases with disseminated disease and serious complications have been described.9

A 46-year-old healthy man presents with a 15-day history of a tender subcutaneous nodule on the dorsum of the right hand that appeared after cleaning his aquarium. He has no fever or systemic symptoms. For 2 weeks he has been taking amoxicillin-clavulanate (Augmentin) and metronidazole (Flagyl), but without an adequate response.

RTEmagicC_Arias-Santiago_MMarinum_F1.gif.gif
Figure 1. A red nodule with central ulceration on the right hand.
RTEmagicC_Arias-Santiago_MMarinum_F2.gif.gif
Figure 2. A nodule on the patient’s forearm with a sporotrichoid distribution. Locoregional lymph nodes were not palpable.
On physical examination, the nodule is painful and has central ulceration (Figure 1). Another two nodules are noted on the forearm, with a sporotrichoid distribution (Figure 2). Locoregional lymph nodes are not palpable.

RTEmagicC_Arias-Santiago_MMarinum_F3.gif.gif
Figure 3. Histologic testing revealed a mixed infiltrate with lymphocytes, neutrophils, and histiocytes (hematoxylin-eosin, × 40).
Laboratory testing (hemography, biochemistry panel, coagulation test, and C-reactive protein) are normal. Routine bacterial cultures of blood and wound drainage are repeatedly negative. Histologic study (Figure 3) reveals signs of acute and chronic inflammation (a mixed infiltrate with lymphocytes, neutrophils, and histiocytes) with abundant bacilli by Ziehl-Neelsen and Giemsa stains.

Q: Which is the most likely diagnosis?

  • Tuberculosis verrucosa cutis
  • Sporotrichosis
  • Nontuberculous mycobacterial infection
  • Leishmaniasis
  • Nocardiosis

A: The correct answer is nontuberculous mycobacterial infection. Mycobacterium marinum was subsequently isolated from culture (Löwenstein-Jensen medium) of wound drainage.

Tuberculosis verrucosa cutis is an indolent, warty plaque that occurs after direct inoculation of M tuberculosis into the skin of someone previously infected with M tuberculosis. It can result from accidental exposure to tuberculous tissue in high-risk groups such as laboratory workers, physicians, and pathologists.

Sporothrix schenckii is the dimorphic fungus that causes sporotrichosis. Activities associated with acquisition of sporotrichosis include gardening (rose-gardener's disease), landscaping, farming, berry-picking, horticulture, and carpentry. The characteristic infection involves suppurating subcutaneous nodules that progress proximally along lymphatic channels (lymphocutaneous sporotrichosis).

Leishmaniasis is a disease caused by the protozoa of the Leishmania species, which is transmitted by the bite of a female sandfly. Initially, the lesion is a small, red papule up to 2 cm in diameter. Over several weeks, the papule becomes darker, develops a crust in the center, and eventually ulcerates to present a typical appearance of an ulcer with raised edges and surrounding dusky red skin.

Nocardia is a genus of filamentous grampositive bacteria that stains acid-fast, just as M marinum does. Nocardia asteroides primarily affects the lungs and can disseminate systemically. However, Nocardia brasiliensis is often associated with sporotrichoid-spreading subcutaneous nodules.

CLINICAL PRESENTATION AND DIAGNOSIS

M marinum is a genus of nontuberculous mycobacteria that usually causes disease in fish but can cause human skin infection by penetrating through a break in the skin. It can spread to deeper structures, resulting in tenosynovitis, arthritis, or osteomyelitis.1 The disease caused by M marinum is sometimes called “swimming pool granuloma” or “fish-tank granuloma.”2

In this case, the patient probably acquired the infection while cleaning his home aquarium,3 so asking about contact with pet fish is very important for the clinical diagnosis.

Lesions are usually localized to the site of the inoculation, with a predilection for areas predisposed to trauma, such as the hands and the fingers, after an incubation period of 2 to 8 weeks. Lesions are self-limited and usually appear as pruriginous bluish-red papules or pustules that may increase in size to form a verrucous or violaceous plaque or nodule. Superficial lesions often undergo central ulceration. Disseminated infection is rare and occurs mainly in patients who are immunocompromised because of renal transplant, systemic lupus erythematosus, chronic steroid therapy, or anti-tumor necrosis factor alpha therapy.4 In such cases, infection can be life-threatening.

On histopathologic study, a nonspecific inflammation consisting of a mixed infiltrate with lymphocytes, neutrophils, and histiocytes is usually observed. Sometimes a granulomatous inflammatory infiltrate mimicking tuberculoid granuloma may be noted. M marinum is an aerobic, nonmotile acid-fast bacillus. It grows at 30°C to 32°C on Löwenstein-Jensen medium in 2 to 5 weeks, but cultures are rarely positive. Polymerase chain reaction and enzyme-linked immunosorbent assays can help identify the organism. Other diagnoses to be considered are leishmaniasis, tuberculosis verrucosa cutis, blastomycosis, histoplasmosis, and sporotrichosis.

TREATMENT

Treatment of M marinum infection is not based on any specific criteria. Spontaneous resolution in 24 to 36 months has been described. 5 Antibiotics are the first-line therapy, and sometimes surgical treatment may be necessary for deeper infection with necrotic tissue. Cryotherapy, electrode therapy, and irradiation have been reported to be effective.

Few studies have been conducted to determine the first-line antibiotic treatment for M marinum infection. In this case, treatment with sulfamethoxazole-trimethoprim6 (Bactrim) resulted in resolution of the lesions in 6 months. Other antibiotics often used as monotherapy are tetracycline, minocycline (Minocin), and doxycycline (Vibramycin) 100 mg twice daily for 3 months.7 However, treatment failure has been described with many antibiotics.

Multidrug therapy is usually prescribed to minimize the risk of resistance. A combination of clarithromycin (Biaxin), streptomycin, and ethionamide (Trecator) has been prescribed successfully.3 Another combined regimen—clarithromycin 500 mg twice a day, rifampin (Rifadin) 600 mg daily, and ethambutol (Myambutol) 25 mg/kg daily—has been shown to be effective.5 Also, rifampin 600 mg daily and ethambutol 15 to 25 mg/kg/day were effective in a patient with a sporotrichoid M marinum infection associated with infliximab (Remicade). 8 The duration of treatment ranged between 2 and 6 months, or up to 2 months after the disappearance of the cutaneous lesions.

Clinical awareness of M marinum infection is important so that the diagnosis can be made and appropriate therapy can be initiated promptly, because some cases with disseminated disease and serious complications have been described.9

References
  1. Lam A, Toma W, Schlesinger N. Mycobacterium marinum arthritis mimicking rheumatoid arthritis. J Rheumatol 2006; 33:817819.
  2. Lewis FM, Marsh BJ, von Reyn CF. Fish tank exposure and cutaneous infections due to Mycobacterium marinum: tuberculin skin testing, treatment, and prevention. Clin Infect Dis 2003; 37:390397.
  3. Dodiuk-Gad R, Dyachenko P, Ziv M, et al. Nontuberculous mycobacterial infections of the skin: a retrospective study of 25 cases. J Am Acad Dermatol 2007; 57:413420.
  4. Streit M, Böhlen LM, Hunziker T, et al. Disseminated Mycobacterium marinum infection with extensive cutaneous eruption and bacteremia in an immunocompromised patient. Eur J Dermatol 2006; 16:7983.
  5. Jogi R, Tyring SK. Therapy of nontuberculous mycobacterial infections. Dermatol Ther 2004; 17:491498.
  6. Alinovi A, Vecchini F, Bassissi P. Sporothricoid mycobacterial infection. A case report. Acta Derm Venereol 1993; 73:146147.
  7. Mahaisavariya P, Chaiprasert A, Khemngern S, et al. Nontuberculous mycobacterial skin infections: clinical and bacteriological studies. J Med Assoc Thai 2003; 86:5260.
  8. Rallis E, Koumantaki-Mathioudaki E, Frangoulis E, Chatziolou E, Katsambas A. Severe sporotrichoid fish tank granuloma following infliximab therapy. Am J Clin Dermatol 2007; 8:385388.
  9. Imakado S, Kojima Y, Hiyoshi T, Morimoto S. Disseminated Mycobacterium marinum infection in a patient with diabetic nephropathy. Diabetes Res Clin Pract 2009; 83:e35e36.
References
  1. Lam A, Toma W, Schlesinger N. Mycobacterium marinum arthritis mimicking rheumatoid arthritis. J Rheumatol 2006; 33:817819.
  2. Lewis FM, Marsh BJ, von Reyn CF. Fish tank exposure and cutaneous infections due to Mycobacterium marinum: tuberculin skin testing, treatment, and prevention. Clin Infect Dis 2003; 37:390397.
  3. Dodiuk-Gad R, Dyachenko P, Ziv M, et al. Nontuberculous mycobacterial infections of the skin: a retrospective study of 25 cases. J Am Acad Dermatol 2007; 57:413420.
  4. Streit M, Böhlen LM, Hunziker T, et al. Disseminated Mycobacterium marinum infection with extensive cutaneous eruption and bacteremia in an immunocompromised patient. Eur J Dermatol 2006; 16:7983.
  5. Jogi R, Tyring SK. Therapy of nontuberculous mycobacterial infections. Dermatol Ther 2004; 17:491498.
  6. Alinovi A, Vecchini F, Bassissi P. Sporothricoid mycobacterial infection. A case report. Acta Derm Venereol 1993; 73:146147.
  7. Mahaisavariya P, Chaiprasert A, Khemngern S, et al. Nontuberculous mycobacterial skin infections: clinical and bacteriological studies. J Med Assoc Thai 2003; 86:5260.
  8. Rallis E, Koumantaki-Mathioudaki E, Frangoulis E, Chatziolou E, Katsambas A. Severe sporotrichoid fish tank granuloma following infliximab therapy. Am J Clin Dermatol 2007; 8:385388.
  9. Imakado S, Kojima Y, Hiyoshi T, Morimoto S. Disseminated Mycobacterium marinum infection in a patient with diabetic nephropathy. Diabetes Res Clin Pract 2009; 83:e35e36.
Issue
Cleveland Clinic Journal of Medicine - 77(8)
Issue
Cleveland Clinic Journal of Medicine - 77(8)
Page Number
512-515
Page Number
512-515
Publications
Publications
Topics
Article Type
Display Headline
Painful red nodule on the right hand
Display Headline
Painful red nodule on the right hand
Sections
Disallow All Ads
Alternative CME
Article PDF Media
Image
Disable zoom
Off

An erythematous plaque on the arm

Article Type
Changed
Tue, 01/16/2018 - 09:30
Display Headline
An erythematous plaque on the arm

A healthy 68-year-old farmer presents with an asymptomatic skin lesion on his right arm that appeared spontaneously without trauma or bite 5 months ago and has grown progressively since then.

RTEmagicC_Arias-Santiago_SquamousCellCarcinoma_F1.gif.gif
Figure 1. An erythematous plaque on the right arm, with an elevated edge, central depression, and hard consistency.
On examination, he has an erythematous plaque that has an elevated edge, a central depression, and a hard consistency (Figure 1). The sensitivity of the lesion is preserved, and no thickened nerves are observed. Local and regional lymph nodes are not palpable. Basic biochemical and blood serologic tests reveal no significant abnormalities.

Q: Which is the most likely diagnosis?

  • Squamous cell carcinoma
  • Cutaneous lymphoma
  • Cutaneous mycobacteriosis
  • Sarcoidosis
  • Lichen planus

RTEmagicC_Arias-Santiago_SquamousCellCarcinoma_F2.gif.gif
Figure 2. Atypical keratinocytes extending from the epidermis to the dermis, with dyskeratosis, intercellular bridges (black arrow), and horn pearl formation (white arrow) (hematoxylin and eosin, × 20).
A: The patient has squamous cell carcinoma. Histopathologic examination of biopsy samples from the lesion showed atypical keratinocytes extending from the epidermis to the dermis, compatible with this diagnosis (Figure 2).

As for the other possibilities:

Cutaneous lymphoma, characterized by atypical proliferation of T or B lymphocytes in the skin, usually presents as a slow-growing indurated nodule or plaque.

Cutaneous mycobacteriosis is caused by infection with Mycobacterium tuberculosis, M leprae, or M marinum. Caseous granulomas are often found on histopathologic study, but special culture and polymerase chain reaction tests are necessary for the diagnosis. Clinically, the lesion is an indurated nodule or plaque that grows slowly, with decreased sensitivity in the case of leprosy, and with a sporotrichoid pattern in the case of M marinum infection.

Our patient underwent a tuberculin skin test, which was negative, as were polymerase chain reaction testing and culture of the biopsy material for mycobacteria.

Sarcoidosis is a systemic disease of unknown cause characterized by noncaseous granulomas in the lungs, mediastinum, lymph nodes, or other sites.1 Cutaneous involvement includes nonspecific clinical forms (eg, nodosum erythema) and other more specific forms (eg, maculopapular, subcutaneous, cicatricial, or lupus pernio variants). This patient has no granulomas on skin biopsy.

Lichen planus is a chronic inflammatory disease of unknown cause that affects the skin, genitalia, mucous membranes, or appendages. It appears as pruritic purple polygonal papules with Wickham striae, usually affecting the wrists and ankles.

A COMMON FORM OF SKIN CANCER

Squamous cell carcinoma and basal cell carcinoma account for 95% of cases of nonmelanomatous skin cancer, and the incidence of squamous cell carcinoma has been increasing.

Long-term exposure to ultraviolet radiation (ie, sunlight), often related to occupation, is the most common cause of squamous cell carcinoma. Other risk factors include thermal injury or burns, ionizing radiation, human papilloma virus infection, immunosuppressive therapy (eg, after organ transplantation2), exposure to chemical carcinogens, and diseases such as xeroderma pigmentosum or oculocutaneous albinism. Unlike basal cell carcinoma, which usually arises de novo, squamous cell carcinoma often arises from precursor lesions, such as actinic keratosis and Bowen disease.

Squamous cell carcinoma commonly manifests as shallow ulcers, often with a keratinous crust and an elevated, indurated border, but also as plaques or nodules, as in this patient. Incipient lesions present as erythematous, firm, keratotic papules, but the lesions on sun-exposed areas such as the head or neck can be pigmented.

The surrounding skin usually shows changes of actinic damage, usually with ulceration, scaling, crusting, or verrucous appearance. When the lesion presents as an erythematous plaque, as in this patient, mycobacterial infection and cutaneous lymphoma should be ruled out. Histologic studies and culture for mycobacteria are necessary for an acute diagnosis. Other conditions in the differential diagnosis are keratoacanthoma, cutaneous metastasis, and sarcoidosis.

The only way to confirm the diagnosis of squamous cell carcinoma is to obtain a specimen by biopsy or surgical excision. Histologic features include atypical keratinocytes extending from the epidermis to the dermis, with dyskeratosis, intercellular bridges, variable central keratinization, and “horn pearls,” depending on how well or how poorly differentiated the tumor.3 Most squamous cell carcinomas arise in solar keratoses, usually at the periphery of the tumor.

Most squamous cell carcinomas are only locally aggressive. Tumors greater than 5 mm in thickness have a 20% risk of metastasis.

TREATMENT OPTIONS

Surgical excision is the most common treatment. Other options are cryotherapy, topical chemotherapy, curettage and electrodesiccation, radiotherapy, and Mohs micrographic surgery. Photodynamic therapy or imiquimod (Aldara) are used only for noninvasive forms.

The possibility of metastasis should be investigated in cases of extensive squamous cell carcinoma, when mucous membranes are affected, and when squamous cell lesions are poorly differentiated.

Our patient’s tumor was removed, with adequate safety margins in surface and depth, and with dermo-epidermal graft closure. The lesion has not recurred at 9 months of follow-up.

References
  1. Hunninghake GW, Costabel U, Ando M, et al. ATS/ERS/WASOG statement on sarcoidosis. American Thoracic Society/European Respiratory Society/World Association of Sarcoidosis and other Granulomatous Disorders. Sarcoidosis Vasc Diffuse Lung Dis 1999; 16:149173.
  2. Rubel JR, Milford EL, Abdi R. Cutaneous neoplasms in renal transplant recipients. Eur J Dermatol 2002; 12:532535.
  3. Cassarino DS, Derienzo DP, Barr RJ. Cutaneous squamous cell carcinoma: a comprehensive clinicopathologic classification—part two. J Cutan Pathol 2006; 33:261279.
Article PDF
Author and Disclosure Information

Salvador Arias-Santiago, MD, PhD
Department of Dermatology, San Cecilio University Hospital, Granada, Spain

José Aneiros-Fernández, MD
Department of Pathology, San Cecilio University Hospital, Granada, Spain

Husein Husein El-Ahmed, MD
Department of Dermatology, San Cecilio University Hospital, Granada, Spain

Pilar Burkhardt-Pérez, MD
Department of Dermatology, San Cecilio University Hospital, Granada, Spain

Ramón Naranjo-Sintes, PhD
Department of Dermatology, San Cecilio University Hospital, Granada, Spain

Address: Salvador Arias-Santiago, MD, Department of Dermatology and Pathology, San Cecilio University Hospital, Av Dr Oloriz 16, Granada 18012, Spain; e-mail salvadorarias@hotmail.es

Issue
Cleveland Clinic Journal of Medicine - 77(6)
Publications
Topics
Page Number
351-352
Sections
Author and Disclosure Information

Salvador Arias-Santiago, MD, PhD
Department of Dermatology, San Cecilio University Hospital, Granada, Spain

José Aneiros-Fernández, MD
Department of Pathology, San Cecilio University Hospital, Granada, Spain

Husein Husein El-Ahmed, MD
Department of Dermatology, San Cecilio University Hospital, Granada, Spain

Pilar Burkhardt-Pérez, MD
Department of Dermatology, San Cecilio University Hospital, Granada, Spain

Ramón Naranjo-Sintes, PhD
Department of Dermatology, San Cecilio University Hospital, Granada, Spain

Address: Salvador Arias-Santiago, MD, Department of Dermatology and Pathology, San Cecilio University Hospital, Av Dr Oloriz 16, Granada 18012, Spain; e-mail salvadorarias@hotmail.es

Author and Disclosure Information

Salvador Arias-Santiago, MD, PhD
Department of Dermatology, San Cecilio University Hospital, Granada, Spain

José Aneiros-Fernández, MD
Department of Pathology, San Cecilio University Hospital, Granada, Spain

Husein Husein El-Ahmed, MD
Department of Dermatology, San Cecilio University Hospital, Granada, Spain

Pilar Burkhardt-Pérez, MD
Department of Dermatology, San Cecilio University Hospital, Granada, Spain

Ramón Naranjo-Sintes, PhD
Department of Dermatology, San Cecilio University Hospital, Granada, Spain

Address: Salvador Arias-Santiago, MD, Department of Dermatology and Pathology, San Cecilio University Hospital, Av Dr Oloriz 16, Granada 18012, Spain; e-mail salvadorarias@hotmail.es

Article PDF
Article PDF

A healthy 68-year-old farmer presents with an asymptomatic skin lesion on his right arm that appeared spontaneously without trauma or bite 5 months ago and has grown progressively since then.

RTEmagicC_Arias-Santiago_SquamousCellCarcinoma_F1.gif.gif
Figure 1. An erythematous plaque on the right arm, with an elevated edge, central depression, and hard consistency.
On examination, he has an erythematous plaque that has an elevated edge, a central depression, and a hard consistency (Figure 1). The sensitivity of the lesion is preserved, and no thickened nerves are observed. Local and regional lymph nodes are not palpable. Basic biochemical and blood serologic tests reveal no significant abnormalities.

Q: Which is the most likely diagnosis?

  • Squamous cell carcinoma
  • Cutaneous lymphoma
  • Cutaneous mycobacteriosis
  • Sarcoidosis
  • Lichen planus

RTEmagicC_Arias-Santiago_SquamousCellCarcinoma_F2.gif.gif
Figure 2. Atypical keratinocytes extending from the epidermis to the dermis, with dyskeratosis, intercellular bridges (black arrow), and horn pearl formation (white arrow) (hematoxylin and eosin, × 20).
A: The patient has squamous cell carcinoma. Histopathologic examination of biopsy samples from the lesion showed atypical keratinocytes extending from the epidermis to the dermis, compatible with this diagnosis (Figure 2).

As for the other possibilities:

Cutaneous lymphoma, characterized by atypical proliferation of T or B lymphocytes in the skin, usually presents as a slow-growing indurated nodule or plaque.

Cutaneous mycobacteriosis is caused by infection with Mycobacterium tuberculosis, M leprae, or M marinum. Caseous granulomas are often found on histopathologic study, but special culture and polymerase chain reaction tests are necessary for the diagnosis. Clinically, the lesion is an indurated nodule or plaque that grows slowly, with decreased sensitivity in the case of leprosy, and with a sporotrichoid pattern in the case of M marinum infection.

Our patient underwent a tuberculin skin test, which was negative, as were polymerase chain reaction testing and culture of the biopsy material for mycobacteria.

Sarcoidosis is a systemic disease of unknown cause characterized by noncaseous granulomas in the lungs, mediastinum, lymph nodes, or other sites.1 Cutaneous involvement includes nonspecific clinical forms (eg, nodosum erythema) and other more specific forms (eg, maculopapular, subcutaneous, cicatricial, or lupus pernio variants). This patient has no granulomas on skin biopsy.

Lichen planus is a chronic inflammatory disease of unknown cause that affects the skin, genitalia, mucous membranes, or appendages. It appears as pruritic purple polygonal papules with Wickham striae, usually affecting the wrists and ankles.

A COMMON FORM OF SKIN CANCER

Squamous cell carcinoma and basal cell carcinoma account for 95% of cases of nonmelanomatous skin cancer, and the incidence of squamous cell carcinoma has been increasing.

Long-term exposure to ultraviolet radiation (ie, sunlight), often related to occupation, is the most common cause of squamous cell carcinoma. Other risk factors include thermal injury or burns, ionizing radiation, human papilloma virus infection, immunosuppressive therapy (eg, after organ transplantation2), exposure to chemical carcinogens, and diseases such as xeroderma pigmentosum or oculocutaneous albinism. Unlike basal cell carcinoma, which usually arises de novo, squamous cell carcinoma often arises from precursor lesions, such as actinic keratosis and Bowen disease.

Squamous cell carcinoma commonly manifests as shallow ulcers, often with a keratinous crust and an elevated, indurated border, but also as plaques or nodules, as in this patient. Incipient lesions present as erythematous, firm, keratotic papules, but the lesions on sun-exposed areas such as the head or neck can be pigmented.

The surrounding skin usually shows changes of actinic damage, usually with ulceration, scaling, crusting, or verrucous appearance. When the lesion presents as an erythematous plaque, as in this patient, mycobacterial infection and cutaneous lymphoma should be ruled out. Histologic studies and culture for mycobacteria are necessary for an acute diagnosis. Other conditions in the differential diagnosis are keratoacanthoma, cutaneous metastasis, and sarcoidosis.

The only way to confirm the diagnosis of squamous cell carcinoma is to obtain a specimen by biopsy or surgical excision. Histologic features include atypical keratinocytes extending from the epidermis to the dermis, with dyskeratosis, intercellular bridges, variable central keratinization, and “horn pearls,” depending on how well or how poorly differentiated the tumor.3 Most squamous cell carcinomas arise in solar keratoses, usually at the periphery of the tumor.

Most squamous cell carcinomas are only locally aggressive. Tumors greater than 5 mm in thickness have a 20% risk of metastasis.

TREATMENT OPTIONS

Surgical excision is the most common treatment. Other options are cryotherapy, topical chemotherapy, curettage and electrodesiccation, radiotherapy, and Mohs micrographic surgery. Photodynamic therapy or imiquimod (Aldara) are used only for noninvasive forms.

The possibility of metastasis should be investigated in cases of extensive squamous cell carcinoma, when mucous membranes are affected, and when squamous cell lesions are poorly differentiated.

Our patient’s tumor was removed, with adequate safety margins in surface and depth, and with dermo-epidermal graft closure. The lesion has not recurred at 9 months of follow-up.

A healthy 68-year-old farmer presents with an asymptomatic skin lesion on his right arm that appeared spontaneously without trauma or bite 5 months ago and has grown progressively since then.

RTEmagicC_Arias-Santiago_SquamousCellCarcinoma_F1.gif.gif
Figure 1. An erythematous plaque on the right arm, with an elevated edge, central depression, and hard consistency.
On examination, he has an erythematous plaque that has an elevated edge, a central depression, and a hard consistency (Figure 1). The sensitivity of the lesion is preserved, and no thickened nerves are observed. Local and regional lymph nodes are not palpable. Basic biochemical and blood serologic tests reveal no significant abnormalities.

Q: Which is the most likely diagnosis?

  • Squamous cell carcinoma
  • Cutaneous lymphoma
  • Cutaneous mycobacteriosis
  • Sarcoidosis
  • Lichen planus

RTEmagicC_Arias-Santiago_SquamousCellCarcinoma_F2.gif.gif
Figure 2. Atypical keratinocytes extending from the epidermis to the dermis, with dyskeratosis, intercellular bridges (black arrow), and horn pearl formation (white arrow) (hematoxylin and eosin, × 20).
A: The patient has squamous cell carcinoma. Histopathologic examination of biopsy samples from the lesion showed atypical keratinocytes extending from the epidermis to the dermis, compatible with this diagnosis (Figure 2).

As for the other possibilities:

Cutaneous lymphoma, characterized by atypical proliferation of T or B lymphocytes in the skin, usually presents as a slow-growing indurated nodule or plaque.

Cutaneous mycobacteriosis is caused by infection with Mycobacterium tuberculosis, M leprae, or M marinum. Caseous granulomas are often found on histopathologic study, but special culture and polymerase chain reaction tests are necessary for the diagnosis. Clinically, the lesion is an indurated nodule or plaque that grows slowly, with decreased sensitivity in the case of leprosy, and with a sporotrichoid pattern in the case of M marinum infection.

Our patient underwent a tuberculin skin test, which was negative, as were polymerase chain reaction testing and culture of the biopsy material for mycobacteria.

Sarcoidosis is a systemic disease of unknown cause characterized by noncaseous granulomas in the lungs, mediastinum, lymph nodes, or other sites.1 Cutaneous involvement includes nonspecific clinical forms (eg, nodosum erythema) and other more specific forms (eg, maculopapular, subcutaneous, cicatricial, or lupus pernio variants). This patient has no granulomas on skin biopsy.

Lichen planus is a chronic inflammatory disease of unknown cause that affects the skin, genitalia, mucous membranes, or appendages. It appears as pruritic purple polygonal papules with Wickham striae, usually affecting the wrists and ankles.

A COMMON FORM OF SKIN CANCER

Squamous cell carcinoma and basal cell carcinoma account for 95% of cases of nonmelanomatous skin cancer, and the incidence of squamous cell carcinoma has been increasing.

Long-term exposure to ultraviolet radiation (ie, sunlight), often related to occupation, is the most common cause of squamous cell carcinoma. Other risk factors include thermal injury or burns, ionizing radiation, human papilloma virus infection, immunosuppressive therapy (eg, after organ transplantation2), exposure to chemical carcinogens, and diseases such as xeroderma pigmentosum or oculocutaneous albinism. Unlike basal cell carcinoma, which usually arises de novo, squamous cell carcinoma often arises from precursor lesions, such as actinic keratosis and Bowen disease.

Squamous cell carcinoma commonly manifests as shallow ulcers, often with a keratinous crust and an elevated, indurated border, but also as plaques or nodules, as in this patient. Incipient lesions present as erythematous, firm, keratotic papules, but the lesions on sun-exposed areas such as the head or neck can be pigmented.

The surrounding skin usually shows changes of actinic damage, usually with ulceration, scaling, crusting, or verrucous appearance. When the lesion presents as an erythematous plaque, as in this patient, mycobacterial infection and cutaneous lymphoma should be ruled out. Histologic studies and culture for mycobacteria are necessary for an acute diagnosis. Other conditions in the differential diagnosis are keratoacanthoma, cutaneous metastasis, and sarcoidosis.

The only way to confirm the diagnosis of squamous cell carcinoma is to obtain a specimen by biopsy or surgical excision. Histologic features include atypical keratinocytes extending from the epidermis to the dermis, with dyskeratosis, intercellular bridges, variable central keratinization, and “horn pearls,” depending on how well or how poorly differentiated the tumor.3 Most squamous cell carcinomas arise in solar keratoses, usually at the periphery of the tumor.

Most squamous cell carcinomas are only locally aggressive. Tumors greater than 5 mm in thickness have a 20% risk of metastasis.

TREATMENT OPTIONS

Surgical excision is the most common treatment. Other options are cryotherapy, topical chemotherapy, curettage and electrodesiccation, radiotherapy, and Mohs micrographic surgery. Photodynamic therapy or imiquimod (Aldara) are used only for noninvasive forms.

The possibility of metastasis should be investigated in cases of extensive squamous cell carcinoma, when mucous membranes are affected, and when squamous cell lesions are poorly differentiated.

Our patient’s tumor was removed, with adequate safety margins in surface and depth, and with dermo-epidermal graft closure. The lesion has not recurred at 9 months of follow-up.

References
  1. Hunninghake GW, Costabel U, Ando M, et al. ATS/ERS/WASOG statement on sarcoidosis. American Thoracic Society/European Respiratory Society/World Association of Sarcoidosis and other Granulomatous Disorders. Sarcoidosis Vasc Diffuse Lung Dis 1999; 16:149173.
  2. Rubel JR, Milford EL, Abdi R. Cutaneous neoplasms in renal transplant recipients. Eur J Dermatol 2002; 12:532535.
  3. Cassarino DS, Derienzo DP, Barr RJ. Cutaneous squamous cell carcinoma: a comprehensive clinicopathologic classification—part two. J Cutan Pathol 2006; 33:261279.
References
  1. Hunninghake GW, Costabel U, Ando M, et al. ATS/ERS/WASOG statement on sarcoidosis. American Thoracic Society/European Respiratory Society/World Association of Sarcoidosis and other Granulomatous Disorders. Sarcoidosis Vasc Diffuse Lung Dis 1999; 16:149173.
  2. Rubel JR, Milford EL, Abdi R. Cutaneous neoplasms in renal transplant recipients. Eur J Dermatol 2002; 12:532535.
  3. Cassarino DS, Derienzo DP, Barr RJ. Cutaneous squamous cell carcinoma: a comprehensive clinicopathologic classification—part two. J Cutan Pathol 2006; 33:261279.
Issue
Cleveland Clinic Journal of Medicine - 77(6)
Issue
Cleveland Clinic Journal of Medicine - 77(6)
Page Number
351-352
Page Number
351-352
Publications
Publications
Topics
Article Type
Display Headline
An erythematous plaque on the arm
Display Headline
An erythematous plaque on the arm
Sections
Disallow All Ads
Alternative CME
Article PDF Media
Image
Disable zoom
Off

Giant nodules on the hands

Article Type
Changed
Wed, 01/17/2018 - 11:51
Display Headline
Giant nodules on the hands

RTEmagicC_Arias-Santiago_TophaceousGout_F1.gif.gif
Figure 1.
A 78-year-old man with hyperuricemia treated with allopurinol (Zyloprim) presented with asymptomatic nodules on both hands (Figure 1), with progressive growth for 2 years.

Q: Which is the most likely diagnosis?

  • Rheumatoid arthritis
  • Nodular osteoarthritis
  • Tophaceous gout
  • Pseudogout
  • Xanthoma tuberosum

A: Tophaceous gout is the diagnosis. This patient’s serum urate level was 9 mg/dL (normal range 4.0–8.0) despite allopurinol therapy, with normal levels of lipids, urea, and creatinine. Polarized light microscopy of aspirated synovial fluid showed monosodium urate crystals, thus confirming the diagnosis.

Rheumatoid arthritis is typically polyarticular and symmetrical and spares the distal interphalangeal joints. Subcutaneous rheumatoid nodules may mimic gouty tophi.

Pseudogout shares some of the features of gout. It results from deposits of calcium pyrophosphate crystals in and around the joints. The diagnosis is made by identifying the crystals on microscopy when calcinosis is seen on x-ray. Tophaceous nodules almost never occur.

Xanthoma tuberosum is associated with hypercholesterolemia, particularly with elevated levels of low-density lipoprotein cholesterol. Lesions occur on pressure areas such as the knees or elbows and vary in size and shape from small papules to firm, lobulated tumors. They are yellow or orange, often with an erythematous halo. They are not associated with chronic proliferative arthritis.

CLINICAL PRESENTATION OF GOUT

Gout is a common metabolic disease characterized by an intermittent course of acute inflammatory arthritis initially affecting one or a few joints. Almost all patients have hyperuricemia, but serum urate levels can be normal or low during an acute attack. On the other hand, many hyperuricemic patients never have a clinical event.

If the hyperuricemia is untreated, some patients develop chronic polyarthritis and nephrolithiasis.1 Inadequate treatment of hyperuricemia may result in chronic tophaceous gout. Although tophaceous gout usually is a sign of long-standing hyperuricemia, tophi can in rare cases be a first symptom of the disorder.2

Even though our patient had been on allopurinol therapy, the dose was not high enough to achieve a serum urate level significantly below the saturation point of urate (about 6.7 mg/dL).

References
  1. Logan JA, Morrison E, McGill PE. Serum uric acid in acute gout. Ann Rheum Dis 1997; 56:696697.
  2. Thissen CA, Frank J, Lucker GP. Tophi as first clinical sign of gout. Int J Dermatol 2008; 47( suppl 1):4951.
Article PDF
Author and Disclosure Information

Salvador Arias-Santiago, MD, PhD
Department of Dermatology, San Cecilio University Hospital, Granada, Spain

José Aneiros-Fernández, MD
Department of Pathology, San Cecilio University Hospital, Granada, Spain

Husein Husein El-Ahmed, MD
Department of Dermatology, San Cecilio University Hospital, Granada, Spain

María-Sierra Girón-Prieto, MD
Department of Dermatology, San Cecilio University Hospital, Granada, Spain

María Antonia Fernández-Pugnaire, PhD
Department of Dermatology, San Cecilio University Hospital, Granada, Spain

Ramón Naranjo-Sintes, PhD
Department of Dermatology, San Cecilio University Hospital, Granada, Spain

Address: Salvador Arias-Santiago, MD, PhD, San Cecilio University Hospital, Av Dr Oloriz 16, Granada 18012, Spain; e-mail salvadorarias@hotmail.es

Issue
Cleveland Clinic Journal of Medicine - 77(4)
Publications
Topics
Page Number
225-229
Sections
Author and Disclosure Information

Salvador Arias-Santiago, MD, PhD
Department of Dermatology, San Cecilio University Hospital, Granada, Spain

José Aneiros-Fernández, MD
Department of Pathology, San Cecilio University Hospital, Granada, Spain

Husein Husein El-Ahmed, MD
Department of Dermatology, San Cecilio University Hospital, Granada, Spain

María-Sierra Girón-Prieto, MD
Department of Dermatology, San Cecilio University Hospital, Granada, Spain

María Antonia Fernández-Pugnaire, PhD
Department of Dermatology, San Cecilio University Hospital, Granada, Spain

Ramón Naranjo-Sintes, PhD
Department of Dermatology, San Cecilio University Hospital, Granada, Spain

Address: Salvador Arias-Santiago, MD, PhD, San Cecilio University Hospital, Av Dr Oloriz 16, Granada 18012, Spain; e-mail salvadorarias@hotmail.es

Author and Disclosure Information

Salvador Arias-Santiago, MD, PhD
Department of Dermatology, San Cecilio University Hospital, Granada, Spain

José Aneiros-Fernández, MD
Department of Pathology, San Cecilio University Hospital, Granada, Spain

Husein Husein El-Ahmed, MD
Department of Dermatology, San Cecilio University Hospital, Granada, Spain

María-Sierra Girón-Prieto, MD
Department of Dermatology, San Cecilio University Hospital, Granada, Spain

María Antonia Fernández-Pugnaire, PhD
Department of Dermatology, San Cecilio University Hospital, Granada, Spain

Ramón Naranjo-Sintes, PhD
Department of Dermatology, San Cecilio University Hospital, Granada, Spain

Address: Salvador Arias-Santiago, MD, PhD, San Cecilio University Hospital, Av Dr Oloriz 16, Granada 18012, Spain; e-mail salvadorarias@hotmail.es

Article PDF
Article PDF

RTEmagicC_Arias-Santiago_TophaceousGout_F1.gif.gif
Figure 1.
A 78-year-old man with hyperuricemia treated with allopurinol (Zyloprim) presented with asymptomatic nodules on both hands (Figure 1), with progressive growth for 2 years.

Q: Which is the most likely diagnosis?

  • Rheumatoid arthritis
  • Nodular osteoarthritis
  • Tophaceous gout
  • Pseudogout
  • Xanthoma tuberosum

A: Tophaceous gout is the diagnosis. This patient’s serum urate level was 9 mg/dL (normal range 4.0–8.0) despite allopurinol therapy, with normal levels of lipids, urea, and creatinine. Polarized light microscopy of aspirated synovial fluid showed monosodium urate crystals, thus confirming the diagnosis.

Rheumatoid arthritis is typically polyarticular and symmetrical and spares the distal interphalangeal joints. Subcutaneous rheumatoid nodules may mimic gouty tophi.

Pseudogout shares some of the features of gout. It results from deposits of calcium pyrophosphate crystals in and around the joints. The diagnosis is made by identifying the crystals on microscopy when calcinosis is seen on x-ray. Tophaceous nodules almost never occur.

Xanthoma tuberosum is associated with hypercholesterolemia, particularly with elevated levels of low-density lipoprotein cholesterol. Lesions occur on pressure areas such as the knees or elbows and vary in size and shape from small papules to firm, lobulated tumors. They are yellow or orange, often with an erythematous halo. They are not associated with chronic proliferative arthritis.

CLINICAL PRESENTATION OF GOUT

Gout is a common metabolic disease characterized by an intermittent course of acute inflammatory arthritis initially affecting one or a few joints. Almost all patients have hyperuricemia, but serum urate levels can be normal or low during an acute attack. On the other hand, many hyperuricemic patients never have a clinical event.

If the hyperuricemia is untreated, some patients develop chronic polyarthritis and nephrolithiasis.1 Inadequate treatment of hyperuricemia may result in chronic tophaceous gout. Although tophaceous gout usually is a sign of long-standing hyperuricemia, tophi can in rare cases be a first symptom of the disorder.2

Even though our patient had been on allopurinol therapy, the dose was not high enough to achieve a serum urate level significantly below the saturation point of urate (about 6.7 mg/dL).

RTEmagicC_Arias-Santiago_TophaceousGout_F1.gif.gif
Figure 1.
A 78-year-old man with hyperuricemia treated with allopurinol (Zyloprim) presented with asymptomatic nodules on both hands (Figure 1), with progressive growth for 2 years.

Q: Which is the most likely diagnosis?

  • Rheumatoid arthritis
  • Nodular osteoarthritis
  • Tophaceous gout
  • Pseudogout
  • Xanthoma tuberosum

A: Tophaceous gout is the diagnosis. This patient’s serum urate level was 9 mg/dL (normal range 4.0–8.0) despite allopurinol therapy, with normal levels of lipids, urea, and creatinine. Polarized light microscopy of aspirated synovial fluid showed monosodium urate crystals, thus confirming the diagnosis.

Rheumatoid arthritis is typically polyarticular and symmetrical and spares the distal interphalangeal joints. Subcutaneous rheumatoid nodules may mimic gouty tophi.

Pseudogout shares some of the features of gout. It results from deposits of calcium pyrophosphate crystals in and around the joints. The diagnosis is made by identifying the crystals on microscopy when calcinosis is seen on x-ray. Tophaceous nodules almost never occur.

Xanthoma tuberosum is associated with hypercholesterolemia, particularly with elevated levels of low-density lipoprotein cholesterol. Lesions occur on pressure areas such as the knees or elbows and vary in size and shape from small papules to firm, lobulated tumors. They are yellow or orange, often with an erythematous halo. They are not associated with chronic proliferative arthritis.

CLINICAL PRESENTATION OF GOUT

Gout is a common metabolic disease characterized by an intermittent course of acute inflammatory arthritis initially affecting one or a few joints. Almost all patients have hyperuricemia, but serum urate levels can be normal or low during an acute attack. On the other hand, many hyperuricemic patients never have a clinical event.

If the hyperuricemia is untreated, some patients develop chronic polyarthritis and nephrolithiasis.1 Inadequate treatment of hyperuricemia may result in chronic tophaceous gout. Although tophaceous gout usually is a sign of long-standing hyperuricemia, tophi can in rare cases be a first symptom of the disorder.2

Even though our patient had been on allopurinol therapy, the dose was not high enough to achieve a serum urate level significantly below the saturation point of urate (about 6.7 mg/dL).

References
  1. Logan JA, Morrison E, McGill PE. Serum uric acid in acute gout. Ann Rheum Dis 1997; 56:696697.
  2. Thissen CA, Frank J, Lucker GP. Tophi as first clinical sign of gout. Int J Dermatol 2008; 47( suppl 1):4951.
References
  1. Logan JA, Morrison E, McGill PE. Serum uric acid in acute gout. Ann Rheum Dis 1997; 56:696697.
  2. Thissen CA, Frank J, Lucker GP. Tophi as first clinical sign of gout. Int J Dermatol 2008; 47( suppl 1):4951.
Issue
Cleveland Clinic Journal of Medicine - 77(4)
Issue
Cleveland Clinic Journal of Medicine - 77(4)
Page Number
225-229
Page Number
225-229
Publications
Publications
Topics
Article Type
Display Headline
Giant nodules on the hands
Display Headline
Giant nodules on the hands
Sections
Disallow All Ads
Alternative CME
Article PDF Media
Image
Disable zoom
Off

Palpable purpura

Article Type
Changed
Mon, 01/15/2018 - 09:24
Display Headline
Palpable purpura

RTEmagicC_Arias_Santiago_Henoch-Schonlein_F1.gif.gif
Figure 1. Erythematous purple papules on the lower legs.
A healthy 47-year-old woman presents with a 3-day history of widespread asymptomatic lesions in the extremities, fever, arthralgias, and mild abdominal pain. A physical examination reveals a symmetric rash in the lower legs with erythematous purple papules. These papules do not blanch when pressure is applied and so are clinically compatible with palpable purpura (Figure 1). Skin biopsy shows leukocytoclastic vasculitis with immunoglobulin A (IgA) deposits in the vascular wall.

Q: Which is the most likely diagnosis?

  • Idiopathic thrombocytopenic purpura
  • Vitamin C deficiency (scurvy)
  • Kaposi sarcoma not related to human immunodeficiency virus (HIV) infection
  • Henoch-Schönlein purpura
  • Polyarteritis nodosa

A: The correct answer is Henoch-Schönlein purpura.

Idiopathic thrombocytopenic purpura is an autoimmune disease caused by specific antibodies against platelet-membrane glycoproteins. It is characterized by thrombocytopenia not explainable by contact with toxic substances or by other causes. Along with nonpalpable purpura, other common signs are epistaxis, gingival bleeding, menorrhagia, and retinal hemorrhage.

Scurvy is an uncommon deficiency of ascorbic acid (vitamin C). The elderly and alcoholics are at higher risk, as they do not take in enough vitamin C in the diet. Patients usually show perifollicular hemorrhages of the skin and mucous membranes, typically petechial hemorrhage or ecchymosis of the gums around the upper incisors. Other cutaneous signs are follicular hyperkeratosis on the forearms, small corkscrew hairs, and sicca syndrome, which is more common in adults.

Non-HIV Kaposi sarcoma usually affects elderly patients, with pink, red, or brown papules or nodules on the legs and, less commonly, on the head and neck. Histopathologic examination shows newly formed irregular blood vessels with an inflammatory infiltrate of plasma cells and lymphocytes; immunohistochemical human herpes virus staining is usually positive.

Polyarteritis nodosa is a systemic vasculitis that affects medium or small arteries with necrotizing inflammation; renal glomeruli and arterioles, capillaries, and venules are unaffected. Skin manifestations include palpable purpura, livedo reticularis, ulcers, and distal gangrene. The condition also usually affects the kidneys, the heart, and the musculoskeletal and nervous systems.

 

 

A SYSTEMIC VASCULITIS

Henoch-Schönlein purpura is a systemic vasculitis affecting the skin, gastrointestinal tract, kidneys, and joints. Palpable purpura and joint pain are the most common and consistent presenting symptoms. The kidneys are affected in about one-third of children and in 60% of adults, and this is the major factor determining the long-term outcome.1

In our patient, laboratory testing that included a complete blood cell count, biochemical testing (including IgA levels), urinary sediment, and coagulation studies showed no abnormalities except elevations of the erythrocyte sedimentation rate and the concentration of C-reactive protein (an acute-phase reactant). These can be normal in some patients. Renal involvement was also not present.

DIAGNOSIS

The diagnosis relies on clinical manifestations. Because Henoch-Schönlein purpura is less common in adults, biopsy plays a more important role in establishing the diagnosis in this age group, and it does this by demonstrating leukocytoclastic vasculitis with a predominance of IgA deposition under immunofluorescence. Recent studies in children showed that an elevated IgA concentration along with reduced IgM levels was associated with a higher rate of severe complications.2 However, depending on the age of the biopsied lesion, IgA may not be detected.

TREATMENT DIRECTED AT SYMPTOMS

Our patient received oral corticosteroids 0.5 mg/kg per day for 20 days, and the lesions resolved by 4 weeks.

Management of Henoch-Schönlein purpura is mainly directed at the symptoms, with oral hydration and nonsteroidal anti-inflammatory drugs. For severe cases, a short course of corticosteroids (0.5–1 mg/kg) may be used.

Although no controlled clinical trial has proven that Henoch-Schönlein purpura responds to corticosteroids, colchicine, or other drugs, corticosteroids are used most often, especially in patients with renal disease. Patients with severe renal insufficiency, abdominal pain, joint involvement, or bleeding should be hospitalized. Plasmapheresis3 has been used in severe cases.

HENOCH-SCHÖNLEIN PURPURA AND MALIGNANCY

During a follow-up evaluation 1 month later, our patient was diagnosed with adenocarcinoma of the breast. This highlights the value of a workup for cancer in adults with cutaneous vasculitis.

Cutaneous vasculitis can represent a paraneoplastic syndrome associated with a malignant tumor. The pathophysiology of this association is unclear, but one proposed mechanism is the exaggerated production of antibodies that react against tumor neoantigens, leading to the formation of immune complexes, or that occasionally recognize endothelial cells because of similarities with tumor antigens. Another theory is that abnormally high levels of inflammatory cytokines are produced by neoplastic cells or in response to decreased immune complex clearance.

Yet another theory is that hyperviscosity of the blood, seen in some cancers, increases the contact time for deposition of immune complexes and causes endothelial damage. Drugs used to treat cancer have also been reported to produce Henoch-Schönlein purpura.4

Although hematologic malignancy is three to five times more common than solid tumors in patients with small-vessel vasculitis, the disease has been associated with solid tumors of the liver, skin, colon, and breast in adults over age 40.5 An evaluation for neoplasm is therefore reasonable in adults with Henoch-Schönlein purpura, as is an evaluation for tumor recurrence or metastasis if the patient has been previously treated for a malignant tumor.

References
  1. Rieu P, Noël LH. Henoch-Schönlein nephritis in children and adults. Morphological features and clinicopathological correlations. Ann Med Interne (Paris) 1999; 150:151159.
  2. Fretzayas A, Sionti I, Moustaki M, Nicolaidou P. Clinical impact of altered immunoglobulin levels in Henoch-Schönlein purpura. Pediatr Int 2009; 51:381384.
  3. Donghi D, Schanz U, Sahrbacher U, et al. Life-threatening or organimpairing Henoch-Schönlein purpura: plasmapheresis may save lives and limit organ damage. Dermatology 2009; 219:167170.
  4. Mitsui H, Shibagaki N, Kawamura T, Matsue H, Shimada S. A clinical study of Henoch-Schönlein purpura associated with malignancy. J Eur Acad Dermatol Venereol 2009; 23:394401.
  5. Maestri A, Malacarne P, Santini A. Henoch-Schönlein syndrome associated with breast cancer. A case report. Angiology 1995; 46:625627.
Article PDF
Author and Disclosure Information

Salvador Arias-Santiago, MD
Department of Dermatology, San Cecilio University Hospital, Granada, Spain

José Aneiros-Fernández, MD
Department of Pathology, San Cecilio University Hospital, Granada, Spain

María Sierra Girón-Prieto, MD
Department of Dermatology, San Cecilio University Hospital, Granada, Spain

María Antonia Fernández-Pugnaire, PhD
Department of Dermatology, San Cecilio University Hospital, Granada, Spain

Ramón Naranjo-Sintes, PhD
Department of Dermatology, San Cecilio University Hospital, Granada, Spain

Address: Salvador Arias-Santiago, MD, San Cecilio University Hospital, Av Dr Oloriz 16, Granada 18012, Spain; e-mail salvadorarias@hotmail.es

Issue
Cleveland Clinic Journal of Medicine - 77(3)
Publications
Topics
Page Number
205-206
Sections
Author and Disclosure Information

Salvador Arias-Santiago, MD
Department of Dermatology, San Cecilio University Hospital, Granada, Spain

José Aneiros-Fernández, MD
Department of Pathology, San Cecilio University Hospital, Granada, Spain

María Sierra Girón-Prieto, MD
Department of Dermatology, San Cecilio University Hospital, Granada, Spain

María Antonia Fernández-Pugnaire, PhD
Department of Dermatology, San Cecilio University Hospital, Granada, Spain

Ramón Naranjo-Sintes, PhD
Department of Dermatology, San Cecilio University Hospital, Granada, Spain

Address: Salvador Arias-Santiago, MD, San Cecilio University Hospital, Av Dr Oloriz 16, Granada 18012, Spain; e-mail salvadorarias@hotmail.es

Author and Disclosure Information

Salvador Arias-Santiago, MD
Department of Dermatology, San Cecilio University Hospital, Granada, Spain

José Aneiros-Fernández, MD
Department of Pathology, San Cecilio University Hospital, Granada, Spain

María Sierra Girón-Prieto, MD
Department of Dermatology, San Cecilio University Hospital, Granada, Spain

María Antonia Fernández-Pugnaire, PhD
Department of Dermatology, San Cecilio University Hospital, Granada, Spain

Ramón Naranjo-Sintes, PhD
Department of Dermatology, San Cecilio University Hospital, Granada, Spain

Address: Salvador Arias-Santiago, MD, San Cecilio University Hospital, Av Dr Oloriz 16, Granada 18012, Spain; e-mail salvadorarias@hotmail.es

Article PDF
Article PDF

RTEmagicC_Arias_Santiago_Henoch-Schonlein_F1.gif.gif
Figure 1. Erythematous purple papules on the lower legs.
A healthy 47-year-old woman presents with a 3-day history of widespread asymptomatic lesions in the extremities, fever, arthralgias, and mild abdominal pain. A physical examination reveals a symmetric rash in the lower legs with erythematous purple papules. These papules do not blanch when pressure is applied and so are clinically compatible with palpable purpura (Figure 1). Skin biopsy shows leukocytoclastic vasculitis with immunoglobulin A (IgA) deposits in the vascular wall.

Q: Which is the most likely diagnosis?

  • Idiopathic thrombocytopenic purpura
  • Vitamin C deficiency (scurvy)
  • Kaposi sarcoma not related to human immunodeficiency virus (HIV) infection
  • Henoch-Schönlein purpura
  • Polyarteritis nodosa

A: The correct answer is Henoch-Schönlein purpura.

Idiopathic thrombocytopenic purpura is an autoimmune disease caused by specific antibodies against platelet-membrane glycoproteins. It is characterized by thrombocytopenia not explainable by contact with toxic substances or by other causes. Along with nonpalpable purpura, other common signs are epistaxis, gingival bleeding, menorrhagia, and retinal hemorrhage.

Scurvy is an uncommon deficiency of ascorbic acid (vitamin C). The elderly and alcoholics are at higher risk, as they do not take in enough vitamin C in the diet. Patients usually show perifollicular hemorrhages of the skin and mucous membranes, typically petechial hemorrhage or ecchymosis of the gums around the upper incisors. Other cutaneous signs are follicular hyperkeratosis on the forearms, small corkscrew hairs, and sicca syndrome, which is more common in adults.

Non-HIV Kaposi sarcoma usually affects elderly patients, with pink, red, or brown papules or nodules on the legs and, less commonly, on the head and neck. Histopathologic examination shows newly formed irregular blood vessels with an inflammatory infiltrate of plasma cells and lymphocytes; immunohistochemical human herpes virus staining is usually positive.

Polyarteritis nodosa is a systemic vasculitis that affects medium or small arteries with necrotizing inflammation; renal glomeruli and arterioles, capillaries, and venules are unaffected. Skin manifestations include palpable purpura, livedo reticularis, ulcers, and distal gangrene. The condition also usually affects the kidneys, the heart, and the musculoskeletal and nervous systems.

 

 

A SYSTEMIC VASCULITIS

Henoch-Schönlein purpura is a systemic vasculitis affecting the skin, gastrointestinal tract, kidneys, and joints. Palpable purpura and joint pain are the most common and consistent presenting symptoms. The kidneys are affected in about one-third of children and in 60% of adults, and this is the major factor determining the long-term outcome.1

In our patient, laboratory testing that included a complete blood cell count, biochemical testing (including IgA levels), urinary sediment, and coagulation studies showed no abnormalities except elevations of the erythrocyte sedimentation rate and the concentration of C-reactive protein (an acute-phase reactant). These can be normal in some patients. Renal involvement was also not present.

DIAGNOSIS

The diagnosis relies on clinical manifestations. Because Henoch-Schönlein purpura is less common in adults, biopsy plays a more important role in establishing the diagnosis in this age group, and it does this by demonstrating leukocytoclastic vasculitis with a predominance of IgA deposition under immunofluorescence. Recent studies in children showed that an elevated IgA concentration along with reduced IgM levels was associated with a higher rate of severe complications.2 However, depending on the age of the biopsied lesion, IgA may not be detected.

TREATMENT DIRECTED AT SYMPTOMS

Our patient received oral corticosteroids 0.5 mg/kg per day for 20 days, and the lesions resolved by 4 weeks.

Management of Henoch-Schönlein purpura is mainly directed at the symptoms, with oral hydration and nonsteroidal anti-inflammatory drugs. For severe cases, a short course of corticosteroids (0.5–1 mg/kg) may be used.

Although no controlled clinical trial has proven that Henoch-Schönlein purpura responds to corticosteroids, colchicine, or other drugs, corticosteroids are used most often, especially in patients with renal disease. Patients with severe renal insufficiency, abdominal pain, joint involvement, or bleeding should be hospitalized. Plasmapheresis3 has been used in severe cases.

HENOCH-SCHÖNLEIN PURPURA AND MALIGNANCY

During a follow-up evaluation 1 month later, our patient was diagnosed with adenocarcinoma of the breast. This highlights the value of a workup for cancer in adults with cutaneous vasculitis.

Cutaneous vasculitis can represent a paraneoplastic syndrome associated with a malignant tumor. The pathophysiology of this association is unclear, but one proposed mechanism is the exaggerated production of antibodies that react against tumor neoantigens, leading to the formation of immune complexes, or that occasionally recognize endothelial cells because of similarities with tumor antigens. Another theory is that abnormally high levels of inflammatory cytokines are produced by neoplastic cells or in response to decreased immune complex clearance.

Yet another theory is that hyperviscosity of the blood, seen in some cancers, increases the contact time for deposition of immune complexes and causes endothelial damage. Drugs used to treat cancer have also been reported to produce Henoch-Schönlein purpura.4

Although hematologic malignancy is three to five times more common than solid tumors in patients with small-vessel vasculitis, the disease has been associated with solid tumors of the liver, skin, colon, and breast in adults over age 40.5 An evaluation for neoplasm is therefore reasonable in adults with Henoch-Schönlein purpura, as is an evaluation for tumor recurrence or metastasis if the patient has been previously treated for a malignant tumor.

RTEmagicC_Arias_Santiago_Henoch-Schonlein_F1.gif.gif
Figure 1. Erythematous purple papules on the lower legs.
A healthy 47-year-old woman presents with a 3-day history of widespread asymptomatic lesions in the extremities, fever, arthralgias, and mild abdominal pain. A physical examination reveals a symmetric rash in the lower legs with erythematous purple papules. These papules do not blanch when pressure is applied and so are clinically compatible with palpable purpura (Figure 1). Skin biopsy shows leukocytoclastic vasculitis with immunoglobulin A (IgA) deposits in the vascular wall.

Q: Which is the most likely diagnosis?

  • Idiopathic thrombocytopenic purpura
  • Vitamin C deficiency (scurvy)
  • Kaposi sarcoma not related to human immunodeficiency virus (HIV) infection
  • Henoch-Schönlein purpura
  • Polyarteritis nodosa

A: The correct answer is Henoch-Schönlein purpura.

Idiopathic thrombocytopenic purpura is an autoimmune disease caused by specific antibodies against platelet-membrane glycoproteins. It is characterized by thrombocytopenia not explainable by contact with toxic substances or by other causes. Along with nonpalpable purpura, other common signs are epistaxis, gingival bleeding, menorrhagia, and retinal hemorrhage.

Scurvy is an uncommon deficiency of ascorbic acid (vitamin C). The elderly and alcoholics are at higher risk, as they do not take in enough vitamin C in the diet. Patients usually show perifollicular hemorrhages of the skin and mucous membranes, typically petechial hemorrhage or ecchymosis of the gums around the upper incisors. Other cutaneous signs are follicular hyperkeratosis on the forearms, small corkscrew hairs, and sicca syndrome, which is more common in adults.

Non-HIV Kaposi sarcoma usually affects elderly patients, with pink, red, or brown papules or nodules on the legs and, less commonly, on the head and neck. Histopathologic examination shows newly formed irregular blood vessels with an inflammatory infiltrate of plasma cells and lymphocytes; immunohistochemical human herpes virus staining is usually positive.

Polyarteritis nodosa is a systemic vasculitis that affects medium or small arteries with necrotizing inflammation; renal glomeruli and arterioles, capillaries, and venules are unaffected. Skin manifestations include palpable purpura, livedo reticularis, ulcers, and distal gangrene. The condition also usually affects the kidneys, the heart, and the musculoskeletal and nervous systems.

 

 

A SYSTEMIC VASCULITIS

Henoch-Schönlein purpura is a systemic vasculitis affecting the skin, gastrointestinal tract, kidneys, and joints. Palpable purpura and joint pain are the most common and consistent presenting symptoms. The kidneys are affected in about one-third of children and in 60% of adults, and this is the major factor determining the long-term outcome.1

In our patient, laboratory testing that included a complete blood cell count, biochemical testing (including IgA levels), urinary sediment, and coagulation studies showed no abnormalities except elevations of the erythrocyte sedimentation rate and the concentration of C-reactive protein (an acute-phase reactant). These can be normal in some patients. Renal involvement was also not present.

DIAGNOSIS

The diagnosis relies on clinical manifestations. Because Henoch-Schönlein purpura is less common in adults, biopsy plays a more important role in establishing the diagnosis in this age group, and it does this by demonstrating leukocytoclastic vasculitis with a predominance of IgA deposition under immunofluorescence. Recent studies in children showed that an elevated IgA concentration along with reduced IgM levels was associated with a higher rate of severe complications.2 However, depending on the age of the biopsied lesion, IgA may not be detected.

TREATMENT DIRECTED AT SYMPTOMS

Our patient received oral corticosteroids 0.5 mg/kg per day for 20 days, and the lesions resolved by 4 weeks.

Management of Henoch-Schönlein purpura is mainly directed at the symptoms, with oral hydration and nonsteroidal anti-inflammatory drugs. For severe cases, a short course of corticosteroids (0.5–1 mg/kg) may be used.

Although no controlled clinical trial has proven that Henoch-Schönlein purpura responds to corticosteroids, colchicine, or other drugs, corticosteroids are used most often, especially in patients with renal disease. Patients with severe renal insufficiency, abdominal pain, joint involvement, or bleeding should be hospitalized. Plasmapheresis3 has been used in severe cases.

HENOCH-SCHÖNLEIN PURPURA AND MALIGNANCY

During a follow-up evaluation 1 month later, our patient was diagnosed with adenocarcinoma of the breast. This highlights the value of a workup for cancer in adults with cutaneous vasculitis.

Cutaneous vasculitis can represent a paraneoplastic syndrome associated with a malignant tumor. The pathophysiology of this association is unclear, but one proposed mechanism is the exaggerated production of antibodies that react against tumor neoantigens, leading to the formation of immune complexes, or that occasionally recognize endothelial cells because of similarities with tumor antigens. Another theory is that abnormally high levels of inflammatory cytokines are produced by neoplastic cells or in response to decreased immune complex clearance.

Yet another theory is that hyperviscosity of the blood, seen in some cancers, increases the contact time for deposition of immune complexes and causes endothelial damage. Drugs used to treat cancer have also been reported to produce Henoch-Schönlein purpura.4

Although hematologic malignancy is three to five times more common than solid tumors in patients with small-vessel vasculitis, the disease has been associated with solid tumors of the liver, skin, colon, and breast in adults over age 40.5 An evaluation for neoplasm is therefore reasonable in adults with Henoch-Schönlein purpura, as is an evaluation for tumor recurrence or metastasis if the patient has been previously treated for a malignant tumor.

References
  1. Rieu P, Noël LH. Henoch-Schönlein nephritis in children and adults. Morphological features and clinicopathological correlations. Ann Med Interne (Paris) 1999; 150:151159.
  2. Fretzayas A, Sionti I, Moustaki M, Nicolaidou P. Clinical impact of altered immunoglobulin levels in Henoch-Schönlein purpura. Pediatr Int 2009; 51:381384.
  3. Donghi D, Schanz U, Sahrbacher U, et al. Life-threatening or organimpairing Henoch-Schönlein purpura: plasmapheresis may save lives and limit organ damage. Dermatology 2009; 219:167170.
  4. Mitsui H, Shibagaki N, Kawamura T, Matsue H, Shimada S. A clinical study of Henoch-Schönlein purpura associated with malignancy. J Eur Acad Dermatol Venereol 2009; 23:394401.
  5. Maestri A, Malacarne P, Santini A. Henoch-Schönlein syndrome associated with breast cancer. A case report. Angiology 1995; 46:625627.
References
  1. Rieu P, Noël LH. Henoch-Schönlein nephritis in children and adults. Morphological features and clinicopathological correlations. Ann Med Interne (Paris) 1999; 150:151159.
  2. Fretzayas A, Sionti I, Moustaki M, Nicolaidou P. Clinical impact of altered immunoglobulin levels in Henoch-Schönlein purpura. Pediatr Int 2009; 51:381384.
  3. Donghi D, Schanz U, Sahrbacher U, et al. Life-threatening or organimpairing Henoch-Schönlein purpura: plasmapheresis may save lives and limit organ damage. Dermatology 2009; 219:167170.
  4. Mitsui H, Shibagaki N, Kawamura T, Matsue H, Shimada S. A clinical study of Henoch-Schönlein purpura associated with malignancy. J Eur Acad Dermatol Venereol 2009; 23:394401.
  5. Maestri A, Malacarne P, Santini A. Henoch-Schönlein syndrome associated with breast cancer. A case report. Angiology 1995; 46:625627.
Issue
Cleveland Clinic Journal of Medicine - 77(3)
Issue
Cleveland Clinic Journal of Medicine - 77(3)
Page Number
205-206
Page Number
205-206
Publications
Publications
Topics
Article Type
Display Headline
Palpable purpura
Display Headline
Palpable purpura
Sections
Disallow All Ads
Alternative CME
Article PDF Media
Image
Disable zoom
Off